91
MA 21 Resolução de Problemas Professor: Sandro Rodrigues Mazorche Grupo: 2 Márcio Eduardo Primo Brasílio Freitas Paulo Geovane Paulo Fernandes Marcelo de Moura Costa

MA21 SOLUÇÕES DAS QUESTÕES DO LIVRO

Embed Size (px)

Citation preview

Page 1: MA21 SOLUÇÕES DAS QUESTÕES DO LIVRO

MA 21 – Resolução de Problemas

Professor: Sandro Rodrigues Mazorche

Grupo: 2

Márcio Eduardo Primo

Brasílio Freitas

Paulo Geovane

Paulo Fernandes

Marcelo de Moura Costa

Page 2: MA21 SOLUÇÕES DAS QUESTÕES DO LIVRO

Capítulo 1 – Soluções dos Exercícios

1. Uma sacola contém meias cujas cores são branca, preta, amarela e azul. Sem olhar para a sacola, qual é a quantidade mínima de meias que precisamos retirar da mesma para garantir pelo menos um par de meias da mesma cor?

Precisamos retirar no mínimo 5 meias. Como o número de meias será maior que o número de cores teremos certeza absoluta que formaremos um par de mesma cor.

2. O pai do padre é filho único de meu pai. O que eu sou do padre?

Como o filho único do meu pai sou eu, então, sou o pai do padre.

3. Numa mesa há 5 cartas:

Q T 3 4 6

Cada carta tem de um lado um número natural e do outro lado uma letra. João afirma: "Qualquer carta que tenha uma vogal tem um número par do outro lado". Pedro provou que João mente virando somente uma das cartas. Qual das 5 cartas foi a que Pedro virou?

Pedro virou a carta 3, mostrando que havia uma vogal escrita no verso.

4. A polícia prende 4 homens, um dos quais é culpado de um furto.

Eles fazem as seguintes declarações:

_ Arnaldo: Bernaldo é o culpável.

_ Bernaldo: Cernaldo é o culpável.

_ Dernaldo: eu não sou culpável.

_ Cernaldo: Bernaldo mente ao dizer que eu sou culpável.

Só uma destas declarações é a verdadeira, quem é culpável pelo furto?

Testando cada uma das afirmações, temos que ter todas as outras falsas.

Suponha então:

(i) Arnaldo fala a verdade. Então: Bernaldo é o culpável. Nesse caso:

1. Benaldo mente. (Falso)

2. Dernaldo fala a verdade. Isso é suficiente para descartar que Arnaldo fale a verdade.

Page 3: MA21 SOLUÇÕES DAS QUESTÕES DO LIVRO

(ii) Bernaldo fala a verdade. Então: Cernaldo é o culpável. Nesse caso:

1. Arnaldo mente. (Falso)

2. Dernaldo fala a verdade. Descartamos ser Bernaldo que falou a verdade.

(iii) Dernaldo fala a verdade. Então: ele não é culpável.

1. Arnaldo pode estar falando verdade ou não

2. Bernaldo pode estar falando verdade ou não

3. Cernaldo pode estar falando verdade ou não. Porém, se a afirmação de Cernaldo for verdadeira a de Bernaldo é necessariamente falsa e vice versa. Ou seja, uma das duas declarações, de Cernaldo ou Bernaldo também será verdadeira, o que descara a possibilidade de que Dernaldo seja o único que fala a verdade.

(iv) Cernaldo fala a verdade. Só restou essa afirmação como possivelmente verdadeira. Nesse caso: Bernaldo mente, e Cernaldo não é culpável.

1. Arnaldo pode estar falando a verdade ou não

2. Bernaldo mente (hipótese de que Cernaldo fale a verdade)

3. Dernaldo pode estar falando a verdade ou não

Concluímos que apenas Cernaldo pode ter falado a verdade, restando que todos outros mentem.

Quem seria o culpável?

Não é Bernaldo e nem Cernaldo, pois Arnaldo e Bernaldo falam mentira.

Dernaldo também mente, então necessariamente ele é o culpável.

5. Numa cidade existe uma pessoa X que sempre mente terças, quintas e sábados e é completamente sincera o resto dos dias da semana. Felipe chega um certo dia na cidade e mantém o seguinte diálogo com a pessoa X:

_ Felipe: Que dia é hoje?

_ X: Sábado.

_ Felipe: Que dia será amanhã?

_ X: Quarta-feira.

Em qual dia da semana foi mantido este diálogo?

X necessariamente está no dia de mentir, pois, não é possível hoje ser sábado e amanhã ser quarta-feira, portanto, pode ser terça, quinta ou sábado. Se fosse terça-feira, a afirmação “Que dia será amanhã?” seria verdadeira. Se fosse Sábado a afirmação “Que dia é hoje?” seria verdadeira. Portanto, o diálogo só pode ter acontecido na quinta-feira.

Page 4: MA21 SOLUÇÕES DAS QUESTÕES DO LIVRO

6. Divida o relógio de parede abaixo em 6 partes iguais de forma tal que a soma das horas que ficam em cada parte seja a mesma.

A soma dos valores: 1 + 2 + ... + 11 + 12 = 78. Dividindo por 6 temos quociente 13. Portanto devemos ter 6 partes de soma 13. Teremos os pares (12, 1); (11, 2); (10, 3); (9, 4); (8, 5) e (6, 7).

Para realizar a divisão em partes iguais deve ser procurada uma disposição geométrica que satisfaça essa condição.

7. João adora Gabriela, que é uma aluna excelente em Matemática. João armou um plano para dar um beijo nela, e descobriu que poderá fazer isso apenas dizendo uma frase. Que frase é essa?

"Se gostas de Matemática deves me dar um beijo, mas se não gostas eu a beijarei".

8. No plano se colocam 187 rodas dentadas do mesmo diâmetro, enumeradas de 1 até 187. A roda 1 é acoplada com a roda 2, a 2 com a 3, : : : , a 186 com a 187 e esta última com a roda 1. Pode tal sistema girar?

As rodas ímpares giram em um sentido e as rodas pares no sentido contrário. Como a roda 187 será acoplada na roda 1 (ambas ímpares) não será possível que o sistema gire.

9. Um canal, em forma quadrada, de 4 metros de largura rodeia um castelo. A ponte do castelo está fechada e um intruso quer entrar no castelo usando duas pranchas de 3,5 metros de comprimento. Será que o intruso consegue?

Sendo: AK = AL temos que AM = 1,75 m

Como AB = 24 m

MB = 24 – 1,75 3,91 m

Como a outra prancha tem 3,5 m não é possível atravessar o fosso.

Page 5: MA21 SOLUÇÕES DAS QUESTÕES DO LIVRO

10. Os números 1, 2, 3, . . . ,99 são escritos no quadro-negro e é permitido realizar a seguinte operação: apagar dois deles e substituí-los pela diferença do maior com o menor. Fazemos esta operação sucessivamente até restar apenas um último número no quadro. Pode o último número que restou ser o zero?

Na sequência 1, 2, ... , 99 excluí-se o 1 e subtraí-se os números consecutivos dois a dois, obteremos 50 números 1 (25 pares) que subtraídos dois a dois, resulta zero.

11. Alguém elege dois números, não necessariamente distintos, no conjunto de números naturais 2, ..., 20. O valor da soma destes números é dado somente a Adriano (A) e o valor do produto dos números é dado unicamente a Karla (K).

_ Pelo telefone A diz a K: _Não é possível que descubras minha soma.

_ Uma hora mais tarde, K diz a A: _Ah! Sabendo disso, já sei quanto vale tua soma!

_ Mais tarde A chama outra vez K e lhe informa: _Poxa, agora eu também conheço teu produto!

Quais números foram eleitos?

Não conseguimos resolver!

Na revista EUREKA! no 23 de 2006 foi publicada uma resolução desse problema pelo Professor Cássio Neri do Instituto de Matemática da Universidade Federal do Rio de Janeiro.

O artigo tem por título "O Problema Impossível" e cita que a forma original foi proposta por Hans Freudenthal e popularizada por Martin Gardner.

Para solucionar, o Professor Cássio usa um algoritmo computacional em linguagem C (disponível em www.labma.ufrj.br/~cassio/f-impossivel.html) e um método não computacional.

Após as considerações, em ambos os casos, ele determina que os números eleitos são 4 e 13.

12. É possível cobrir um tabuleiro de xadrez com 31 dominós onde removemos as casas dos vértices superior esquerdo e inferior direito?

Não pois as casas retiradas estão na mesma diagonal, são casas de mesma cor.

13. Num saco encontram-se 64 moedas leves e 64 moedas pesadas. É possível separar duas moedas de pesos diferentes com 7 pesagens?

Inicialmente dividimos as moedas em dois grupos de 64 moedas. Colocamos cada grupo nos pratos da balança.

Page 6: MA21 SOLUÇÕES DAS QUESTÕES DO LIVRO

Na segunda pesagem escolha as 64 moedas do prato mais leve, caso fiquem desequilibrados, ou qualquer grupo se ficaram em equilíbrio e separe em dois grupos de 32 moedas em cada prato da balança.

Para realizar a terceira pesagem, proceda como na segunda, separando agora 16 moedas em cada prato da balança.

Na quarta pesagem escolha um dos grupos de 16 e divida-o em dois grupos de 8 moedas para cada prato da balança.

Chegando na quinta pesagem divida as oito moedas obtidas na pesagem anterior e divida em dois grupos de quatro moedas para cada prato.

Agora na sexta pesagem divida um dos grupos de 4 moedas em dois grupos de 2 moedas para cada prato.

Na sétima, e última pesagem, escolha um grupo de 2 moedas da sexta pesagem e coloque uma moeda em cada prato. Se houver desequilíbrio as moedas são as que estão nos pratos. Se houver equilíbrio as moedas de pesos diferentes são as outras duas separadas anteriormente.

14. Quantas vezes precisamos dobrar um papel de 1mm de espessura para que a altura da pilha chegue da Terra à Lua?

Cada dobra efetuada nos fornecerá uma nova altura que corresponderá ao dobro da altura anterior. A sequência formada será uma PG de razão 2. Portanto teremos que encontrar um termo na PG que seja igual à distância da Terra à Lua.

Seja X a distância, em milímetros, da Terra à Lua teremos então:

12.1 nX 2:2nX nX 2.2

nX 2log).2(log

2log.log2log nX 2log

log2log Xn

2log

log1

Xn

15. Descubra o erro da prova da afirmação abaixo:

Afirmação: Três é igual a dois.

Seja x um número diferente de zero. Temos que:

3x – 3x = 2x – 2x

Colocando x – x em evidência, temos que:

3(x – x) = 2(x – x).

Cancelando x – x em ambos os lados, obtemos que 3 = 2.

O erro está no cancelamento de x – x. Nessa passagem é efetuada uma divisão por zero.

Page 7: MA21 SOLUÇÕES DAS QUESTÕES DO LIVRO

Capítulo 2 – Soluções dos Exercícios

1. Observe as multiplicações a seguir:

(a) 12.345.679 x 18 = 222.222.222

(b) 12.345.679 x 27 = 333.333.333

(c) 12.345.679 x 54 = 666.666.666

Para obter 999.999.999 devemos multiplicar 12.345.679 por quanto?

Solução:

O algarismo que desejo repetir resulta da multiplicação do próprio por 9, ou seja:

12.345.679 x 9.n = nnn.nnn.nnn (com 0 < n < 10, sendo n um número natural)

Como desejo repetir o algarismo 9, temos que multiplicar por 81.

2. Outro dia ganhei 250 reais, incluindo o pagamento de horas.extras. O salário (sem horas extras) excede em 200 reais o que recebi pelas horas extras. Qual é o meu salário sem horas extras?

Solução: Denominando S = salário e HE = hora extra, teremos:

(1) S + HE = 250 (2) S = HE + 200 Substituindo a relação (2) em (1), teremos: 200 + HE + HE = 250 HE = 25 S = 225 O meu salário sem horas extras é de R$225,00.

3. Uma torneira A enche sozinha um tanque em 10 h, uma torneira B enche o mesmo tanque sozinha em 15 h. Em quantas horas as duas torneiras juntas encherão o tanque? Solução: Temos que:

⇒ T = 6 horas

4. O dobro de um número, mais a sua terça parte, mais a sua quarta parte somam 31. Determine o número. Solução: Denominando de x o número a ser descoberto:

24x + 4x + 3x = 372

31x = 372

x = 12

Page 8: MA21 SOLUÇÕES DAS QUESTÕES DO LIVRO

5. Uma certa importância deve ser dividida entre 10 pessoas em partes iguais. Se a partilha fosse feita somente entre 8 dessas pessoas, cada uma destas receberia R$5.000,00 a mais. Calcule a

importância. Solução: Denominando: P = parte que cada pessoa irá receber; T = total a ser dividido.

4T + 200.000 = 5T

T = 200.000

A importância é de R$200.000,00. 6. Roberto disse a Valéria: “Pense um número, dobre esse número, some 12 ao resultado, divida o novo resultado por 2. Quanto deu?” Valéria disse “15” ao Roberto, que imediatamente revelou o número original que Valéria havia pensado. Calcule esse número. Solução: Seja x o número pensado por Valéria, Temos que:

1º) Passo: 2x

2º) Passo: 2x + 12

3º) Passo:

4º) Passo: x + 6 = 15

Logo; x = 9

7. Por 2/3 de um lote de peças iguais, um comerciante pagou R$8.000,00 a mais do que pagaria pelos 2/5 do mesmo lote. Qual o preço do lote todo?

Solução:

Denominando de x o valor total do lote, teremos:

10x – 120.000 = 6x

x = 30.000

O valor total do lote é de R$30.000,00

8. Determine um número real a para que as expressões 3a+6/8 e 2a+10/6 sejam iguais.

Solução:

Page 9: MA21 SOLUÇÕES DAS QUESTÕES DO LIVRO

9a +18 = 8a + 40

a = 22 9. Se você multiplicar um número real x por ele mesmo e do resultado subtrair 14, você vai obter o quíntuplo do número x. Qual é esse número? Solução:

Temos: {

. * +

10. Eu tenho o dobro da idade que tu tinhas quando eu tinha a tua idade. Quando tu tiveres a minha idade, a soma das nossas idades será de 45 anos. Quais são as nossas idades? Solução: Denominando: Minha idade = y Sua idade = x A diferença entre as idades = k Teremos então:

(1) y = 2.(x – k) (2) x + k + y + k = 45

Substituindo (1) em (2) 3x = 45 x = 15

Como y = 2x – 2k e y – x = k, teremos y = k + x

y = k + 15 k + 15 = 30 – 2k

k = 5 y = 5 + 15 = 20

Nossas idades são: eu tenho 20 anos e você 15 anos. 11. Um homem gastou tudo o que tinha no bolso em três lojas. Em cada uma gastou 1 real a mais do que a metade do que tinha ao entrar. Quanto o homem tinha ao entrar na primeira loja? Solução: Denominando de x o valor com o qual o homem tenha entrado no local. Resolvendo o problema de trás para frente, teremos: 3º local: x/2 – 1 = 0, x = 2, logo, ele entrou com R$2,00. 2º local: x/2 – 1 = 2, x = 6, logo, ele entrou com R$6,00. 1º local: x/2 – 1 = 6, x = 14, logo, ele entrou com R$14,00. Ele entrou na primeira loja com R$14,00.

Page 10: MA21 SOLUÇÕES DAS QUESTÕES DO LIVRO

12. Com os algarismos x, y e z formam-se os números de dois algarismos xy e yx, cuja soma é o número de três algarismos zxz. Quanto valem x, y e z? Solução: Os algarismos são xy e yx, e a soma é zxz. Considerando os algarismos como x, y, z Temos que (10x + y) + (10y + x) = 100z + 10x + z e 11x + 11y = 101z + 10x (ou ainda x + 11y = 101z) O número da forma zxz precisa estar entre 101 e 98 + 89 = 187, ou seja 101 < zxz <181. Como o número é 11x + 11y = 11(x + y), temos que o número é um múltiplo de 11. Ou seja, as soluções possíveis são os múltiplos de 11 entre 101 e 181, ou seja: 110, 121, 132, 143, 154, 165, 176. Dentre esses, apenas o 121 tem o algarismo da centena igual ao da unidade. Logo z = 1, x = 2. Precisamos encontrar y. Ora, x + 11y = 101z, como já enunciamos. Fazendo as substituições: 2 + 11y = 101 => 11y = 99 => y = 9 Os algarismos são x = 2, y = 9, z = 1 13. Quantos são os números inteiros de 2 algarismos que são iguais ao dobro do produto de seus algarismos? Solução: Assumindo que os algarismos que compõe o número sejam denominados de x(algarismo das dezenas) e y(algarismo das unidades), teremos:

10x + y = 2xy

O dobro do produto dos algarismos implica que o número seja par, logo, o algarismo das unidades só poderá ser: 0,2,4,6 ou 8

Porém podemos excluir y = 0 pois o produto dos algarismos seria zero. Se y = 2, temos que 10x + 2 = 4x, solução descartada. Se y = 4, temos que 10x + 4 = 8x, solução descartada.

Se y = 6, temos que 10x + 6 = 12x, então x = 3. Se y = 8, temos que 10x + 8 = 16x, solução descartada.

O único número que atende o problema é o 36.

14. Obter dois números consecutivos inteiros cuja soma seja igual a 57. Solução: Denominando um número de x e seu posterior de x + 1, teremos:

x + (x + 1) = 57 x = 28

Os números que atendem a solução do problema são 28 e 29.

Page 11: MA21 SOLUÇÕES DAS QUESTÕES DO LIVRO

15. Qual é o número que, adicionado ao triplo do seu quadrado, vale 14? Solução: Chamando o número de x, temos: x + 3x2 = 14 x = – 7/3 ou x = 2. 16. O produto de um número positivo pela sua terça parte é igual a 12. Qual é esse número? Solução: Denominando o número de x, teremos:

x2 = 36

x = 6, mas como o enunciado diz que o número é positivo, x = 6. 17. Determine dois números consecutivos ímpares cujo produto seja 195. Solução: Denominando um número ímpar por 2x + 1, e seu consecutivo por 2x + 3, teremos:

(2x + 1).(2x + 3) = 195 4x2 + 8x – 192 = 0

x' = – 8 ou x’’ = 6, fazendo as devidas substituições teremos: Ou os números são 13 e 15 ou – 15 e – 13.

18. A diferença entre as idades de dois irmãos é 3 anos e o produto de suas idades é 270. Qual é a idade de cada um? Solução: Chamando as idades de x e y, temos duas equações

{

y2 + 3y – 270 = 0 P = – 270 S = – 3 y = 15 ou y = –18 Como x e y representam idades consideramos apenas o valor positivo. Portanto y = 15 e x = 18. Então as idades dos irmãos são 15 e 18 anos 19. Calcule as dimensões de um retângulo de 16 cm de perímetro e 15 cm2 de área. Solução: Denominando as dimensões de x e y, temos pelo perímetro e pela área que:

(pelo perímetro) 2x + 2y = 16

Page 12: MA21 SOLUÇÕES DAS QUESTÕES DO LIVRO

x + y = 8 y = x – 8

(pela área) x.y = 15 (substituindo y) x.(x – 8) = 15

x2 – 8x – 15 = 0 x’ = 5 ou x’’ = – 3

Como estamos tratando de dimensões geométricas, usaremos somente o valor positivo, logo: x = 5 e y = 3

20. A diferença de um número e o seu inverso é 8/3 . Qual é esse número? Solução: Denominando o número de x, teremos:

3x2 – 3 = 8x

3x2 – 8x – 3 = 0 x' = 3 ou x’’ = – 1/3

Questão 21: A soma de dois números é 12 e a soma de seus quadrados é 74. Determine os dois números. Solução: Chamando os dois números de x e y temos:

{

Da primeira equação, temos que:

(x + y)2 = (12)2 x2 + y2 + 2xy = 144

(substituindo pela segunda equação) 74 + 2xy = 144 2xy = 70 xy = 35 x = 35/y

(substituindo na primeira equação)

35 + y2 = 12y y2 – 12y + 35 = 0 y’ = 5 ou y’’ = 7

Se y = 5, x = 7 ou se y = 7, x = 5 Os números são 7 e 5.

Page 13: MA21 SOLUÇÕES DAS QUESTÕES DO LIVRO

22. Um pai tinha 30 anos quando seu filho nasceu. Se multiplicarmos as idades que possuem hoje, obtém-se um produto que é igual a três vezes o quadrado da idade do filho. Quais são as suas idades? Solução: Denominando de P = idade do pai, e F = idade do filho, temos que:

P – F = 30 PF = 3F2 → P = 3F

(substituindo na primeira equação) 3F – F = 30 F = 15 e P = 45

O pai tem 45 anos enquanto que o filho tem 15 anos. 23. Os elefantes de um zoológico estão de dieta juntos. Num período de 10 dias devem comer uma quantidade de cenouras igual ao quadrado da quantidade que um coelho come em 30 dias. Em um dia os elefantes e o coelho comem juntos 1.444 kg de cenoura. Quantos quilos de cenoura os elefantes comem em 1 dia? Solução: Denominando de E = quantidade que os elefantes devem comer em um dia e C = quantidade que um coelho deve comer em um dia, teremos:

{ ( )

⇒ {

10C = 14440 – 900C2 90C2 + C – 1444 = 0

C’ = 4 C’’< 0 (descartado) E = 1440 kg

Um elefante come 1440 kg de cenoura por dia. Questão 24: Sejam α1 e α2 as raízes do polinômio ax2 + bx + c, com a ≠ 0. Calcule as seguintes expressões, em função de a, b e c:

(a)

(b) √ √

(c) √ √

Solução:

a)

b) ( ) √

√ √ √

c) (√ √

) √ √ √ √

Page 14: MA21 SOLUÇÕES DAS QUESTÕES DO LIVRO

√ √

√√

25. O número – 3 é a raiz da equação x2 – 7x – 2c = 0. Nessas condições, determine o valor do coeficiente c. Solução:

Se –3 é raiz, então: p(– 3) = 0 (– 3)2 – 7.( – 3) – 2.c = 0

c = 15

26. Encontre o polinômio p(x) = 2x4 + bx3 + cx2 + dx+ e que satisfaz a equação p(x) = p(1 - x). Solução:

p(x) = 2x4 + bx3 + cx2 + dx+ e p(x – 1) = 2(x – 1)4 + b(x – 1)3 + c(x – 1)2 + d(x – 1)+ e

p(x – 1) = 2x4 + (– 8 – b)x3 + (12 + 3b + c)x2 + (– 8 – 3b – 2c – d)x + (2 + b + c + d + e) Pela identidade de polinômios, temos que: p(x) ≡ (p – 1), comparando os coeficientes de mesmo grau

iremos concluir que:

b = – 4, c = t (t ℝ), d = 2 – t, e ℝ

O polinômio procurado será: p(x) = 2x4 – 4x3 + tx2 + (2 – t )x+ e Questão 27: (OBM) Dois meninos jogam o seguinte jogo. O primeiro escolhe dois números inteiros diferentes de zero e o segundo monta uma equação do segundo grau usando como coeficientes os dois números escolhidos pelo primeiro jogador e 1.998, na ordem que quiser (ou seja, se o primeiro jogador escolhe a e b o segundo jogador pode montar a equação 1.998x2 + ax + b = 0 ou ax2 + 1.998x + b = 0 etc.) O primeiro jogador é considerado vencedor se a equação tiver duas raízes racionais diferentes. Mostre que o primeiro jogador pode ganhar sempre. Solução: Inicialmente veja que, se num polinômio p(x) = anx

n + … + a1x + a0, tivermos an + … a1 + a0 = 0, teremos p (1) = 0 e 1 é raiz de p. Dessa forma se o primeiro jogador escolhe b = – (1998 + a), 1 será raiz da equação do segundo grau que o seu oponente irá montar. Se um polinômio tem coeficientes inteiros (na verdade vale para coeficientes racionais) e possui uma

raiz irracional do tipo a + b r (r não é quadrado perfeito), então a – b r também é raiz. Ou seja, as

raizes irracionais vêm aos pares. No caso de uma equação de segundo grau, e coeficientes inteiros, ambas as raizes são irracionais, ou ambas são racionais. No nosso caso, como 1 já é raiz, a outra raiz será racional. Basta ver então, apenas se 1 não é raiz multipla (pois queremos que as raizes sejam distintas). Para isso basta escolher a adequadamente.

Se o primeiro jogador escolher os números a = n1998, e b = – (n + 1) 1998, n 2 N, ele ganha. (1 não será raiz múltipla e a equação terá duas raizes racionais distintas.)

Obs. É possivel obter soluções com a + b + 1998 0, por exemplo com

{a, b} = {2040, – 5478} (solução obtida com o auxílio de um computador.)

Page 15: MA21 SOLUÇÕES DAS QUESTÕES DO LIVRO

28. (OBM) Mostre que a equação x2 + y2 + z2 = 3xyz tem infinitas soluções onde x;y;z são números inteiros. Solução: Escolhemos uma solução para a equação. Tomamos por exemplo o terno (1,1,1)

x² + 1² + 1² = 3.x.1.1 ou seja, x=1 ou x= 2

Então, além de (1,1,1), o terno (2,1,1) também é solução da equação. Considerando agora o terno (2,1,1) teremos então por exemplo que 2² + 1² + z² = 3.2.1.z, ou seja z=1 ou z=5. Então além de (2,1,1), o terno (2,1,5) também é solução. De modo geral escolhendo uma terna e escolhendo um dos três valores, sempre acharemos outro valor que satisfaz também a equação. Ou seja de um terno (x0 , y0 , z0) sempre obteremos um outro terno (x1, y1, z1). Escolhendo x0 teremos: x² + y0² + z0 ² = 3.x. y0 .z0 x² - 3.x. y0 .z0 + y0² + z0 ² = 0 Com essa equação do 2º grau, sabemos então que a soma das raízes é dada por s = 3. y0 .z0 Então x1 > x0 se x0 < 3/2 . y0 .z0 e caso contrário x0 > 3/2 . y0 .z0 . Com isso garantimos que a segunda raiz sempre será maior que a primeira que foi substituída na equação, e consequentemente, existirá infinitas soluções para a equação. 29. (Gazeta Matemática, Romênia) Considere a equação a2x2 (b2 - 2ac)x + c2 = 0; onde a;b e c são

números inteiros positivos. Se n é tal que p(n) = 0, mostre que n é um quadrado perfeito. Solução: Temos que:

0)( np 0)2( 2222 cncabna

Observe que essa é uma equação do segundo grau na incógnita n . Para resolver essa equação

encontremos o seu discriminante:

)4(44444])2([ 22242222242222 acbbcabbcacacabbcaacb

Daí, podemos concluir que )4( 22 acbb é um quadrado perfeito, pois Nn é raiz de )(xp , ou seja,

0)( np e a , b e c são inteiros positivos o que nos garante que acb 42 também é um quadrado

perfeito, já que 2b também o é. Mas acb 42 é o discriminante da equação do segundo grau

02 cxbxa , onde suas raízes a

bx

2'

e

a

bx

2"

, devido ao valor de , são

números racionais.

Voltando para a resolução da equação inicial, 0)2( 2222 cncabna , temos que:

a

c

a

cabb

a

b

a

cabbcab

a

cabbcabn

2

4

2

42

2

)4(2 2

2

22

2

222

Page 16: MA21 SOLUÇÕES DAS QUESTÕES DO LIVRO

Mas,

"' xxa

b , "' xx

a

c , '

2

42

xa

cabb

e "

2

42

xa

cabb

, onde 'x e "x são, como já

vimos, as raízes da equação 02 cxbxa .

Portanto, fazendo as devidas substituições, temos que:

"'"')'("'')"'(' 2 xxxxxxxxxxn 2)'(' xn

Ou:

"'"')"("'")"'(" 2 xxxxxxxxxxn 2)"(" xn

Logo, como n é um número natural que é igual ao quadrado de números racionais, então esses

números racionais tem que ser inteiros e n é, portanto, um quadrado perfeito.

Questão 30: (Gazeta Matemática, Romênia) Sejam a, b Z. Sabendo que a equação (ax – b)2 + (bx – a)2 = x, tem uma raiz inteira, encontre os valores de suas raízes. Solução: (ax – b)2 + (bx – a)2 = x a2x2 – 2abx + b2 + b2x2 – 2abx + a2 = x (a2 + b2)x2 – (4ab + 1)x + a2 + b2 = 0 Se o produto é igual a 1 e uma as raízes é inteira, então elas são inversas e:

4ab + 1 ≥ 2.(a2 + b2)

2a2 + 2b2 – 4ab ≥ 1

2.(a – b)2 ≥ 1

⇒ ⇒

31. (Gazeta Matemática, Romênia) Resolva a equação:

*

+

Obs.: [x] é o menor inteiro maior ou igual a x. Solução:

Page 17: MA21 SOLUÇÕES DAS QUESTÕES DO LIVRO

Por definição de [a], teremos que: x , 2x2 e x2 + 1 .

A expressão acima equivale a dizer que x é o quociente da divisão euclidiana de 2x2 por x2 + 1, isto é,

equivale a dizer que: r {0, 1, ..., x, ..., x2}, tal que:

2x2 = (x2 + 1).x + r, daí podemos afirmar

– x3 + 2x2 – x = r x.( – x2 + 2x – 1) = r

mas temos que x | r, então concluímos que r {0, x, 2x, ..., x2}, o que nos permite dizer que r = .x

com {0, 1, ..., x}. Segue que

x.( – x2 + 2x – 1) = .r

x.( – x2 + 2x – (1+)) = 0

Segue daí que x = 0 ou que ( – x2 + 2x – (1+)) = 0. Da segunda igualdade, termos que √ ,

mas como x e {0, 1, ..., x}, = 0, e daí, termos x = 1 como a segunda solução.

32. Demonstrar que: (a) n4 + 4 não é primo se n > 1; (b) generalize, mostrando que n4 + 4n não é primo, para todo n > 1.

a) não é primo se n > 1

Solução:

Considere a igualdade de Sophie Germain

a4 + 4b4 = (a2 + 2b2 + 2ab)( a2 + 2b2 - 2ab)

Tomando a = n e b = 1, temos:

n4 + 4 = (n2 + 2 + 2n)( n2 + 2 - 2n), assim, n4 + 4 é composto.

b) Generalize, mostrando que n4 + 4n não é primo, para todo n > 1.

Solução:

Se n for par, segue que n4 e 4n são pares, logo n4 + 4n não é primo.

Se n for ímpar, n é da forma 2m + 1, então, substituindo temos:

(2m + 1)4 + 42m + 1

(2m + 1)4 + 4 42m

(2m + 1)4 + 4 24m

(2m + 1)4 + 4

Tomando a = 2m + 1 e b = 2m , o resultado é conseqüência da igualdade de Sophie Germain.

Page 18: MA21 SOLUÇÕES DAS QUESTÕES DO LIVRO

Questão 33: Para fazer 12 bolinhos, preciso exatamente de 100 g de açúcar, 50 g de manteiga, meio litro de leite e 400 g de farinha. Qual a maior quantidade desses bolinhos que serei capaz de fazer com 500 g de açúcar, 300 g de manteiga, 4 litros de leite e 5 kg de farinha? Solução: Para fazer 12 bolinhos: 100g de Açúcar 50g de Manteiga 0,5 l de Leite 400g de Farinha. Com: 500g de Açucar conseguimos fazer 60 bolinhos 50g de Manteiga conseguimos fazer 72 bolinhos 0,5l de Leite conseguimos fazer 96 bolinhos 400g de Farinha conseguimos fazer 150 bolinhos Portanto o açúcar é o ingrediente limite e podemos fazer 60 bolinhos no máximo. 34. Dadas as frações

qual é maior? Solução:

Denominando x = , x +1 = e x + 2 = , teremos uma expressão a qual iremos supor uma afirmação, de que a primeira fração é menor do que a segunda.

O que é verdade, logo, a primeira fração é menor do que a segunda!

35. Achar o maior valor inteiro positivo de n tal que n200 < 5300: Solução: Aplicando as propriedades das potências, temos que:

( ) ( )

Como a base é positiva , segue que:

n2 < 53

n < √

n < 11,2

Page 19: MA21 SOLUÇÕES DAS QUESTÕES DO LIVRO

Como n , temos que o maior valor inteiro positivo será quando n = 11.

36. Achar o menor valor inteiro positivo de n tal que

Solução:

1 + 2 + 3 + ... + n > 55

( )

n2 + n – 110 > 0

n > 0 (positivo), logo, n = 11.

37. Nove cópias de certas notas custam menos de R$ 10,00 e dez cópias das mesmas notas (com o mesmo preço) custam mais de R$ 11,00. Quanto custa uma cópia das notas? Solução: Denominando de x = valor de uma cópia, teremos que:

9x < 10 → x < 10/9 10x > 11 → x > 11/10

Analisando as desigualdades, x < 1,111... e x > 1,10, iremos concluir que x = 1,11.

Uma cópia custa R$1,11. 38. Se enumeram de 1 até n as páginas de um livro. Ao somar estes números, por engano um deles é somado duas vezes, obtendo-se o resultado incorreto: 1.986. Qual é o número da página que foi somado duas vezes? Solução:

Considere k o número que foi somado duas vezes. O valor de k é um número natural maior ou igual a 1 e menor ou igual a n.

Page 20: MA21 SOLUÇÕES DAS QUESTÕES DO LIVRO

Assim:

i) 1986 – (1 + 2 + ... + n) ≥ 1

1986 – [( )

- ≥ 1

– n2 – n ≥ 3970

n2 + n – 3970 ≤ 0

Resolvendo a inequação, encontramos:

- 63,5 ≤ n ≤ 62,5

ii) 1986 – (1 + 2 + ... + n) ≤ n

1986 – [( )

- ≤ n

– n2 – n ≤ 3972

n2 + 3n – 3972 ≥ 0

Resolvendo a inequação, encontramos:

n ≤ - 64,5 e n ≥ 61,5

As duas inequações apresentam o seguinte conjunto solução para n natural

61,5 ≤ n ≤ 62,5,

Segue que o único valor de compreendido nesse intervalo é 62.

Assim, o número de páginas do livro é 62 páginas

A soma das páginas seria:

1 + 2 + 3 + ... + 62 = ( )

.

Como havia somado 1986, logo: 1986 – 1953 = 33. A página somada duas vezes, foi a página 33. 39. Determine os valores de a para os quais a função quadrática ax2 – ax + 12 é sempre positiva. Solução: Para que a função seja sempre positiva deve obedecer:

a > 0 e < 0

= a2 – 4.a.12 < 0 0 < a < 48.

Para uma equação ter apenas resultados positivos é necessário que delta seja menor que zero, e o coeficiente do termo quadrático positivo. 40. Ache os valores de x para os quais cada uma das seguintes expressões é positiva:

( )

( )

( )

a) Solução: Pelo numerador temos que teremos uma imagem positiva para x > 0 e imagem negativa para x < 0. Enquanto que para o denominador teremos uma imagem positiva para qualquer valor de x (discriminante menor que zero) Fazendo o estudo dos sinais, veremos que o valor que satisfará a condição do problema será:

S = { x ℝ / x > 0}

Page 21: MA21 SOLUÇÕES DAS QUESTÕES DO LIVRO

b) Solução: Pelo numerador temos que teremos uma imagem positiva para x > 3 e imagem negativa para x < 3. Enquanto que para o denominador teremos uma imagem positiva para x > – 1 e teremos uma imagem negativa para x < – 1. Fazendo o estudo dos sinais, veremos que o valor que satisfará a condição do problema será:

S = { x ℝ / x < – 1 ou x > 3}

c) Solução: Pelo numerador temos que teremos uma imagem positiva para x < – 1 ou x > 1 e imagem negativa para – 1 < x < 1. Enquanto que para o denominador teremos uma imagem positiva para x < 0 ou x > 3 e teremos uma imagem negativa para 0 < x < 3. Fazendo o estudo dos sinais, veremos que o valor que satisfará a condição do problema será:

S = { x ℝ / x < – 1 ou 0 < x < 1 ou x > 3}

41. Resolve r a equação: [x]{x}+ x = 2{x}+ 10, onde [x] denota a parte inteira de x. Por exemplo, [2;46] = 2 e [5;83] = 5. O número {x} é chamado parte fracionária de x e é definido por {x} = x - [x]. Solução:

Considere x = [x] + {x} e substituindo na equação, temos:

|x|{x} + [x] + {x} = 2{x} + 10

|x|{x} + [x] – {x} = 10

{x} ( [x] – 1) = 10 – [x]

{x} = , -

, - , da expressão temos que considerar que:

i) 0 < {x} ≤ 1, visto que a parte fracionária tem que ser um valor entre 0 e 1.

ii) [x] ≤ 10, pelo fato de que, se fosse maior que 10, teríamos uma parte fracionária negativa.

iii) 10 – [x] ≤ [x] – 1, isto é, o numerador tem que ser menor ou igual ao denominador.

Resolvendo essa inequação temos:

– [x] – [x] ≤ – 1 – 10

– 2 [x] ≤ – 11

[x] ≥ 5,5 , mas como [x] tem que ser inteiro, então [x] ≥ 6

Assim, 6 ≤ [x] ≤ 10.

Substituindo em {x} = , -

, - , obtemos:

Page 22: MA21 SOLUÇÕES DAS QUESTÕES DO LIVRO

i) Para [x] = 6 , {x} =

iv) Para [x] = 9 , {x} =

ii) Para [x] = 7 , {x} =

=

v) Para [x] = 10 , {x} =

iii) Para [x] = 8 , {x} =

Assim, a solução é : 6, 8; 7,5;

; 9,125 e 10

42. Mostre que entre os retângulos com um mesmo perímetro, o de maior área é um quadrado. Solução:

Considere um retângulo cujos lados medem x e y. Temos que a área, A, desse retângulo é dada por

e seu perímetro, p, é dado por

.

Logo, .

/

.

E mais,

. Portanto, para A máximo temos x = y.

43. Entre todos os triângulos isósceles com perímetro p fixado, ache as dimensões dos lados daquele que possui a maior área.

Solução:

Denominando cada um dos dois lados congruentes de a e o terceiro lado de b e o perímetro de p, teremos que:

p = a + a + b

p = 2a + b

Usando a fórmula de Herón temos que a área de um triângulo com perímetro p é dada pela expressão

.

/ .

/.

/

No caso específico do triângulo isósceles, ela ficará

.

/ .

/ .

/

Analisando uma parte da fórmula: .

/ .

/ .

/,

Temos que MG ≤ MA, logo,

√.

/ .

/ .

/

O que nos permite concluir que

Page 23: MA21 SOLUÇÕES DAS QUESTÕES DO LIVRO

(√.

/ .

/ .

/

)

(

)

(√.

/ .

/ .

/

)

.

/

.

/ .

/ .

/

O maior valor da desigualdade que o primeiro membro atingirá será quando ele for igual ao segundo membro, logo, substituindo na fórmula de Herón, teremos:

Logo a maior área possível é

√ , a qual é atingida quando

.

/ .

/ ⇔

Ou seja, quando o triângulo for eqüilátero.

44. (OBM Júnior 1993)

É dada uma equação do segundo grau x2 + ax + b = 0, com raízes inteiras a1 e b1. Consideramos a equação do segundo grau x2 + a1x + b1 = 0. Se a equação x2 + a1x + b1 = 0 tem raízes inteiras a2 e b2, consideram s a equação x2 + a2x+ b2 = 0. Se a equação x2 +a2x+b2 = 0 tem raízes inteiras a3 e b3,

consideramos a equação x2 + a3x+ b3 = 0. E assim por diante . Se encontramos uma equação com

< 0 ou com raízes que não sejam inteiros, encerramos o processo. Por exemplo, se começamos com a equação x2 = 0 podemos continuar o processo indefinidamente. Pede-se:

(a) Determine uma outra equação que, como x2 = 0, nos permita continuar o processo indefinidamente;

(b) Determine todas as equações do segundo grau completas a partir das quais possamos continuar o processo indefinidamente.

Seja P0(x) = x2 + a0x + b0 = 0 uma equação com raízes inteiras a1 e b1.

Vamos definir a equação Pi(x) = x2 + aix + bi = 0 como sendo a equação obtida no i-ésimo passo do procedimento descrito pelo enunciado, ou seja, ai e bi são as raízes inteiras da equação Pi – 1(x) = x2 + ai - 1x + bi - 1 = 0, para i = 1, 2, ... .

Responderemos agora os dois itens.

Page 24: MA21 SOLUÇÕES DAS QUESTÕES DO LIVRO

a) Solução:

Seja P0(x) = x2 + kx = 0, onde k .

Como se vê facilmente, podemos obter a partir de P0(x) = 0 as equações:

Pi(x) = x2 + (– 1)ikx = 0, i = 1, 2, ... .

b) Solução:

Nosso próximo passo será determinar todas as equações que nos possibilitem continuar o processo indefinidamente.

Vimos no item (a) que as equações x2 + kx = 0, k , têm essa propriedade, por isso suponhamos b0

≠ 0 em P0(x) = x2 + a0x + b0 = 0. Evidentemente bi - 1 = ai .bi, ou seja, bi divide bi – 1, o que mostra que, para todo i inteiro positivo, bi divide b0.

Além disso, b0 possui um número finito de divisores, pois b0 ≠ 0.

Daí resulta que, se b0 ≠ 0 e o processo continua indefinidamente, pelo menos um dos divisores bj de b0 deve aparecer infinitas vezes como termo independente das equações do processo.

Digamos que bj volte aparecer como termo independente de uma equação Pk(x) = x2 + akx + bk = 0 para algum k > j.

Mas como bi divide bi – 1 para todo i inteiro positivo, temos |bi – 1| ≥ |bi| e daí |bj| ≥ |bj+1|≥ ... ≥ |bk| = |bj|, o

que mostra que |bi| = |bj| para i = j + 1, j + 2, ..., k, onde bi { – bj, bj} para todo i > j.

Há dois casos a considerar:

, ( )

( )

e

, ( )

( )

No primeiro caso, observemos que é raiz de ( ) , o que implica que a outra raiz é – 1 e

( ) .

Se o termo independente de ( ) é , então 1 é raiz de ( ), o que implica que ( )

⇒ , o que é absurdo pois divide b0.

Se o termo independente de ( ) é , então – 1 é raiz de ( ), o que mostra que ( )

⇒ ⇒ ( ) ⇒ ( ) , o que é uma equação cujas raízes

não são inteiras.

Vemos portanto que neste caso não é possível continuar o processo.

Uma análise inteiramente análoga a do primeiro caso permite-nos concluir, no segundo caso,

( ) que é uma equação onde uma das raízes é 1 ou – 1.

Page 25: MA21 SOLUÇÕES DAS QUESTÕES DO LIVRO

Se a raiz for – 1, então ( ) , o que é absurdo.

Se a raiz for 1, então ( ) ⇒ .

Logo ( ) , cujas raízes são 1 e – 2.

Assim podemos fazer ( ) para i ≥ j + 2, e repetir este procedimento indefinidamente.

É imediata a verificação de que ( ) também implica que ( )

para 0 ≤ i ≤ j.

Segue que ( ) é a única equação com termo independente não nulo a partir da qual podemos repetir o processo um número arbitrário de vezes.

Page 26: MA21 SOLUÇÕES DAS QUESTÕES DO LIVRO

Capítulo 3 – Soluções dos Exercícios

1. Encontre o resto que deixa

(a) 2001. 2002 . 2003 . 2004 + 20052 quando é dividido por 7;

O produto 2001. 2002 . 2003 . 2004 é divisível por 7 pois 2002 0 mod 7.

Como 2005 3 mod 7 temos que 20052 32 mod 7 2 mod 7.

Logo o resto será igual a 2.

(b) 2100 quando é dividido por 3;

Sabemos que 22 1 mod 3 e 502100 22 portanto 2100 150 mod 3 1 mod 3.

Então o resto será igual a 1.

(c) 2856 3412371 quando é dividido por 111.

Sabemos que 12371 50 mod 111; 123712 58 mod 111; 123718 584 mod 111 46 mod 111.

Como 111mod16111mod461237112371 77856 .

Portanto devemos encontrar o resto da divisão de 2828503416 por 111.

Temos que 111mod58502 ; como 14228 5050 então 111mod5850 1428 .

Sendo 111mod34582 logo 111mod345858 77214 .

Se 437 34.3434 e 111mod10343 e 111mod7344 então 111mod10.7347 .

Portanto temos um resto igual a 70.

2. Provar que o número n5 + 4n é divisível por 5 para qualquer número natural n.

Adicionando e subtraindo 5n à expressão n5 + 4n teremos então n5 + 4n + 5n – 5n = n5 – n + 5n.

Pelo pequeno teorema de Fermat temos que nn 55 e n55 .

Sabemos que se ba e ca então cba .

Portanto nn 45 5 .

Page 27: MA21 SOLUÇÕES DAS QUESTÕES DO LIVRO

3. Prove que se n é ímpar

(a) n3 – n é divisível por 24;

Seja 1..13 nnnnnP um número inteiro com n ímpar.

i) Sabemos que o produto de dois números consecutivos é múltiplo de 2, então P é divisível por 2.

ii) Sendo n ímpar temos que 1n e 1n são pares então P é divisível por 22.

iii) O produto de três números consecutivos é divisível por 3, então P é divisível por 3.

iv) Todo número divisível por 2 e 3 é divisível por 6, então P é divisível por 6.

v) Se P é múltiplo de 6dee22então será múltiplo de 24.

(b) n2 – 1 é divisível por 8;

Seja 1.112 nnnP um número inteiro com n ímpar.

Sendo 12 kn , para k Z temos que )1.(422.2112.11212 kkkkkknP .

i) Se k é par temos que P = 4k = 8m.

ii) Se k é ímpar k + 1 é par então temos P = 8m.

Portanto P será sempre múltiplo de 8.

(c) n2 + (n + 2)2 + (n + 4)2 + 1 é divisível por 12.

Seja 142222 nnnP um número inteiro com n ímpar.

Desenvolvendo temos 116844 222 nnnnnP = 74321123 22 nnnn .

Sendo 12 kn , para k Z temos que 71241232

kkP

Page 28: MA21 SOLUÇÕES DAS QUESTÕES DO LIVRO

7481443 2 kkkP 121243 2 kkP 3343 2 kkP

3312 2 kkP mP 12 , m Z, logo P é múltiplo de 12.

4. O número 21093 – 2 é divisível por 10932?

Como 1093 é primo e 11093,2 , temos, pelo Pequeno Teorema de Fermat, que 221093 1093 .

Sabemos que 12.222 10921093 .

Da aritmética dos restos temos que 2modmod pbapba pppp , com a, b, p N e p primo.

Fazendo p = 1093 temos que 21093109310931093 1093mod1093mod baba .

Temos que 1211093.2218737 p logo 1093mod11093.237

pp mod1237 2227 mod123 pp 2214 mod1443 ppp

Como 22 mod04 pp então 214 mod143 pp (1)

1115111093.1516384214 p

pp mod1115214 22214 mod11152 pp 2228 mod1213302252 ppp

228 mod1213302 pp multiplicando por 32 temos 2282 mod108929702.3 pp .

Como 2970p = 1093p + 1876p + p; 1089 = 1093 – 4; 22 mod0 pp e sendo p = 1093 temos:

2282 mod418762.3 pp que dividido por 4 resulta em 2262 mod14692.3 pp

277262 mod14692.3 pp 2718214 mod14692.3 pp

Desenvolvendo o binômio e excluindo as congruências 2mod0 p restará:

218214 mod132832.3 pp 218214 mod141093.32.3 ppp

2218214 mod1432.3 ppp 218214 mod142.3 pp (2)

Usando as expressões (1) e (2) temos 2182 mod12 p 266182 mod12 p

21092 mod12 p multiplicando por 2 temos que 21093 mod22 p .

Logo 221093 1093 .

5. Prove que 19999941234567890 é divisível por 333331.

Considere 19999941234567890P .

Page 29: MA21 SOLUÇÕES DAS QUESTÕES DO LIVRO

Podemos escrever

1234567889

1

1234567890 999994.19999941999994k

kP .

Logo MPk

k .999993999994.9999931234567889

1

, sendo M N.

Como 333331.3999993 então SMP .333331.333331.3 , sendo S N.

Portanto P é múltiplo de 333331.

6. O número N = 42005 + 20054 é primo?

450142004420054 4.420054.4200542005 N .

Fazendo 2005a e 5014b , segue pela desigualdade de Sophie Germain que N é composto.

7. Demonstre que o número zeros2006

001...1000 é composto.

1000...001 = zeros2007

00...1000 + 1 = 1102007

1102007 = 33669 110

Fazendo 33669 110 bea temos que 2233 . bbaababa .

Como 2233 . bbaababa é composto então zeros2006

001...1000 também é composto.

8. Utilizando o fato de que o resto de um quadrado quando dividido por 4 só pode ser 0 ou 1, dê uma outra solução para o problema do Exemplo 3.54.

Seja P = aabb um número natural quadrado perfeito a {1, 2, ..., 9} e a b.

Temos que P = 4q + 0 ou P = 4q + 1.

Para P = 4mod0aabb temos que b = 0; b = 4 ou b = 8.

Para b = 8, P não é quadrado perfeito, pois nenhum quadrado perfeito termina em 8.

Page 30: MA21 SOLUÇÕES DAS QUESTÕES DO LIVRO

Para b = 0, P não é quadrado perfeito, pois teríamos 100.aaP e não existe nenhum quadrado

perfeito da forma aa.

Para b = 4, P = aa44 temos que 10030 P . Por tentativa temos que 882 = 7744, portanto a = 7.

Para P = 4mod1aabb 1114 b , logo b = 3 ou b = 7. Nestes casos P também não é quadrado

perfeito, pois nenhum quadrado perfeito termina em 3 ou em 7.

9. Dados três inteiros, x, y, z, tais que x2 + y2 = z2, mostre que x e y não são ambos ímpares e que xy é múltiplo de 6.

I) Suponha por absurdo x e y ímpares. Sendo x = 2t + 1 e y = 2v + 1 e {t, v} Z.

Como x2 + y2 = z2 então 222 144144 zvvtt 222 24 zvvtt 4m + 2 = z2,

mostrando que z2 é par logo z também é par.

Seja z= 2u com u Z, então z2 = 4u2.

Portanto 4m + 2 = 4u2 dividindo por 2 temos parímpar

um 2212 o que é um absurdo.

II) Mostrar que xy é múltiplo de 3

Suponha x e y não múltiplos de 3. Logo z = 3k, k Z.

Seja 2,1,33 212211 rrcomrqyerqx .

Como x2 + y2 = z2 temos que 22

222

2

2

2

111

2

1 96969 krrqqrrqq então 2

2

2

13 rr .

Como x e y não são ambos ímpares e xy é múltiplo de 3 então xy é múltiplo de 6.

10. Demonstre que o quadrado de um inteiro é da forma 8n ou 8n+1 ou 8n + 4.

Seja m um número inteiro. Podemos escrever m = 4k; m = 4k + 1; m = 4k + 2 ou m = 4k + 3.

Se m = 4k então m2 = 16k2 = 8(2k2) = 8n.

Se m = 4k + 1 então m2 = (4k + 1)2 = 16k2 + 8k + 1 = 8 (2k2 + k) + 1 = 8n + 1.

Se m = 4k + 2 então m2 = (4k + 2)2 = 16k2 + 16k + 4 = 8 (2k2 + 2k) + 4 = 8n + 4.

Se m = 4k + 3 então m2 = (4k + 3)2 = 16k2 + 24k + 9 = 16k2 + 24k + 8 + 1 = 8(2k2 + 3k + 1) + 1 = 8n + 1.

Outra solução:

Page 31: MA21 SOLUÇÕES DAS QUESTÕES DO LIVRO

Seja p = 4q + r a divisão de p por 4. Assim 22222 28816 rqrqrqrqp .

Logo 8mod22 rp , como r {0, 1, 2, 3} então r2 {0, 1, 4}.

Portanto np 82 ou 182 np ou 482 np .

11. Três números primos p; q e r, maiores que 3, formam uma progressão aritmética, ou seja, q = p + d e r = p + 2d. Prove que d é divisível por 6.

Para provar que d é divisível por 6, devemos mostrar que 2|d e 3|d.

Como todos os primos maiores que 3 são ímpares e, supondo que d seja ímpar, temos que q é par, pois, a soma de dois números ímpares resulta em um número par, o que é um absurdo, pois, q é primo. Assim, d é par, isto é, 2|d.

Supondo que d não seja divisível por 3, então d é da forma 3k + 1 ou 3k + 2, com k N.

Para d = 3k + 1, verificamos que:

p = p q = p + d = p + 3k + 1 r = p + 2(3k + 1) = p + 3(2k) + 2

Para d = 3k + 2, verificamos que:

p = p q = p + d = p + 3k + 2 r = p + 2(3k + 2) = p + 3(2k + 1) + 1

Assim, pelo Lema dos restos, na divisão por 3 os valores de q e r são da forma p + 1 e p + 2.

Logo, sendo p, q e r três números consecutivos, ou seja, p, p + 1 e p + 2, então, pelo menos um deles é múltiplo de 3, mostrando que 3| d.

Portanto, como 2|d e 3|d, então 6|d.

12. Demonstrar que existem infinitos números primos da forma 4m + 3 e da forma 6m + 5, onde m Z.

Um número inteiro P qualquer pode ser inscrito como rqP 4 , com r {0, 1, 2, 3}. Se P é primo

então r = 1 ou r = 3.

Suponha que existam finitos primos na forma 4m + 3 e seja A o conjunto de todos os primos dessa

forma, ou seja nppppNmmA ,...,,5,2,14 210 .

Observe que o conjunto A é fechado multiplicativamente.

Suponhamos, por absurdo, que existam apenas um número finito de primos p1 < ... < pk da forma

34 m , com m > 1. O número P = 4(p1.p2....pk) + 3 não é divisível por nenhum dos números primos 3,

p1, ..., pk e, portanto, sua decomposição em fatores primos só pode conter primos da forma 4m + 1, o que é uma contradição, pois P é da forma 4m + 3.

Page 32: MA21 SOLUÇÕES DAS QUESTÕES DO LIVRO

Considere um número natural S qualquer. Podemos escrevê-lo na forma rm 6 , 5,...,1,0r .

Se o número S é primo então 5,1r .

Suponhamos, por absurdo, que existam apenas um número finito de primos p1 < ... < pk da forma

56 m , com m > 1. O número S = 6(p1.p2....pk) + 5 não é divisível por nenhum dos números primos 5,

p1, ..., pk e, portanto, sua decomposição em fatores primos só pode conter primos da forma 6m + 1.

Como o conjunto B = {6n + 1; nN} é fechado multiplicativamente, então S é da forma 6m + 1, o que é uma contradição. Logo existem infinitos números primos da forma 6m + 5.

13. Encontrar o último dígito dos números (resto da divisão por 10)

(a) 19892005

Sabemos que 100222005 1989.19891989 .

10mod91989 e 10mod110mod8110mod91989 22

10mod110mod11989 100210022 .

10mod910mod1.91989.1989198910022205

Logo o dígito da unidade é 9.

(b) 777777 + 250

10mod18134 e 10mod727

10mod7777 10mod710mod1.73.273.3343777764425632592593777

10mod225 10mod41024222 1010550

Logo 10mod110mod472777 50777 , então o algarismo da unidade é 1.

(c) 1 + 22 + 32 + ... + 20052.

Sabemos que 10mod11991...21111 ; 10mod21992...22122 , ou seja números de mesmo final possuem mesmos restos mod 10 e sabemos, também, que todo número quadrado só termina em 0, 1, 4, 5, 6 ou 9.

Dividindo 2000 em grupos de 10 temos 200 grupos.

Portanto temos

Page 33: MA21 SOLUÇÕES DAS QUESTÕES DO LIVRO

i) de 1 até 2000 temos 10mod0900045.2000149656941.200

ii) de 2001 até 2005 temos 10mod52554941

Portanto o algarismo da unidade é 5.

14. Prove que a soma dos quadrados de cinco números consecutivos não é um quadrado perfeito.

Sejam 2;1;;1;2 nnnnn números naturais consecutivos.

Somando os quadrados 222222112 nnnnn temos 105 2 n .

Se n é par (n = 2k) temos 24 ' k . Se n é ímpar (n = 2k + 1) temos 34 '' k .

Como um quadrado perfeito é da forma k4 ou 14 k .

Portanto a soma dos quadrados de cinco números consecutivos não é um quadrado perfeito.

15. Prove que 100...000500...0001100100

zeroszeros

não é um cubo perfeito.

Se n é inteiro podemos escrever kknqn ,93 3 ou kknqn ,1913 3

ou

kknqn ,8923 3.

Seja 100...000500...0001100100

zeroszeros

X então podemos escrever 110.510 101202 X .

Pelo lema dos restos temos que 9mod110202 ; 9mod1.510.5 101 e 9mod11 .

Logo X deixa resto 1 + 5 + 1 = 7 quando dividido por 9, portanto X não é cubo perfeito.

16. Seja b um inteiro positivo. Enuncie e prove o critério de divisibilidade por b no sistema de numeração de base b.

Proposição: Se ZaaaaPbnn 011... com nibai ,...,2,1,0,1,...,2,1,0 , então Pb

00 a .

() Seja P um número escrito na base b, logo 01

1

1 ... abababaP n

n

n

n

.

Podemos escrever 01

2

1

1 )....( aabababP n

n

n

n

.

Pela proposição 3.3 temos que Pb , logo 01

2

1

1 )....( aabababb n

n

n

n

.

Page 34: MA21 SOLUÇÕES DAS QUESTÕES DO LIVRO

É fato que )....( 1

2

1

1 abababb n

n

n

n

então b também deve dividir 0a . Como 10 0 ba

temos necessariamente que 00 a .

() Seja bababaP n

n

n

n 1

1

1 ...

um número em base b, logo

qbabababP n

n

n

n .)....( 1

2

1

1

Pb

17. Prove que os números

(a) n

n

1...

3

1

2

11 , com n > 1,

Considere p o maior número primo tal que p < n, então,

np

npnppnpnpn

......321

1......21......11...21.........431......32

Fazendo

e

logo .

Como p não divide npp ...11...321 , que é uma parcela de m, então pk também não divide

npp ...11...321 portanto, n não é inteiro.

(b) 12

1...

5

1

3

1

nn , com n > 0, não são inteiros.

Considere p o maior número primo tal que p < n.

Então,

12......753

12......3...12...22...53...12......7312......75

np

npnppnpnpn

Fazendo

1......21......11...21.........431......32 npnppnpnpm

nppk ...11...21pk

mn

Page 35: MA21 SOLUÇÕES DAS QUESTÕES DO LIVRO

12......3...12...22...53...12......7312......75 npnppnpnpm

e 12...22...753 nppk então pk

mn .

Como p não divide 12...22...753 npp , que é uma parcela de m, então pk também

não divide 12...22...753 npp , portanto, n não é inteiro.

18. Considere o polinômio 0

1

1 ...)( anananp m

m

m

m

de grau m > 1 com coeficientes inteiros e

nN. Prove que p(n) é um número composto para infinitos valores de n.

Sugestão: Use o fato de que existe aN tal que 1)( ap e mostre que divide akp , para

todo k Z.

Sabemos que )(ap e temos que 0

1

1 ...)( aaaaaap m

m

m

m

.

Calculando akp em 0

1

1 ...)( anananp m

m

m

m

temos:

0

1

1 ...)()()( aakaakaakp m

m

m

m

que, após desenvolver os binômios de cada

parcela, restará kkkaaaaaakpmmm

m

m

m

......)(1

0

1

1 .

Como aN, a primeira parcela representa )(ap e como k Z, a segunda parcela representa

uma soma de potências de que pode ser escrita como .Q , pois é múltiplo de .

Então .1.)( TQQakp

19. Dizemos que um conjunto An formado por n inteiros positivos escritos no sistema binário (base 2) é regular se, para qualquer s inteiro não negativo a quantidade de números de An que contemplam 2s na representação binária é par. Dizemos que An é irregular se, pelo menos para algum s, este número é ímpar. Demonstre que um sistema irregular pode se converter em regular excluindo-se apenas um único elemento do mesmo, e, um sistema regular pode se converter em irregular excluindo-se qualquer um dos seus elementos.

20. Seja n um inteiro positivo. Demonstrar que todos os coeficientes do desenvolvimento do binômio

de Newton (a + b)n são ímpares se, e somente se, n é da forma 12 s.

Page 36: MA21 SOLUÇÕES DAS QUESTÕES DO LIVRO

21. Prove que se (x0, y0) é uma solução da equação diofantina linear ax – by = 1, então a área do

triângulo cujos vértices são (0, 0), (b, a) e (x0, y0) é 2

1.

Se (x0, y0) é solução da equação ax – by = 1, então ax0 – by0 = 1.

Sabemos que a área de uma região triangular, no plano, pode ser dada por DA .2

1 , sendo D o

determinante formado com as coordenadas dos vértices.

Então

1

1

100

2

100

ab

yxA = 00

2

1byax = 1

2

1 =

2

1

22. Qual é a menor distância possível entre dois pontos (x1, y1) e (x2, y2), com coordenadas inteiras, situados sobre a reta definida pela equação diofantina ax + by = c?

Temos que cbyaxcbyax 2211 e

Então 2211 byaxbyax logo 2121 xxayyb 2121 xxb

ayy .

Como a distância entre dois pontos no plano é dada por 221

2

21 yyxxd temos, então que

2

21

2

21

xx

b

axxd 2

212

22

21 xxb

axxd

2

22

21 1b

axxd

2

22

21b

baxxd .

Se (x1, y1) (x2, y2) e as coordenadas são inteiras temos d 0 e 121 xx .

A distância mínima ocorre quando 121 xx então

2

22

minb

bad .

Page 37: MA21 SOLUÇÕES DAS QUESTÕES DO LIVRO

Capítulo 4- Soluções dos Exercícios

1. Seja C um conjunto formado por cinco pontos de coordenadas inteiras no plano. Prove que o ponto médio de algum dos segmentos com extremos em C tem também coordenadas inteiras. Solução: Seja C = { P1, P2, P3, P4, P5 } o conjuntos dos 5 pontos de coordenadas inteiras no plano. Temos as seguintes possibilidades: P1= { par, par}; P2= { par ímpar}; P3= {ímpar, par} e P4= {ímpar, ímpar}. O ponto médio entre dois pontos A e B é dado por M( xM, yM) , onde:

e

Observe que para o ponto médio tenha coordenadas inteiras xA e xB, é necessário que sejam ambos pares ou ambos ímpares, o mesmo ocorre com yA e yB. Dessa forma, os pontos P1, P2, P3, P4, P5 representam os pombos, e a paridade de suas coordenadas representam as casas, onde não pode haver outra possibilidade. Pelo P.C.P., P5 terá a paridade de suas coordenadas igual a um dos pontos P1, P2, P3, P4 . Portanto, o ponto médio de algum dos segmentos com extremidades em C tem também coordenadas inteiras. 2. O conjunto dos dígitos 1, 2, ..., 9 é dividido em três grupos. Prove que o produto de alguns dos grupos deve ser maior que 71. Solução: Sejam 1, 2, 3, os produtos dos elementos do grupo 1, do grupo 2 e do grupo 3, respectivamente. Suponha que:

1 ≤71

2 ≤71 3 ≤71

Daí, como 1 > 0, 2 > 0, 3 > 0, temos:

362880 = 1 ∙ 2 ∙ 3 ≤357911

ou seja,

362880 ≤357911 (absurdo!!!)

Portanto, o produto de alguns dos grupos deve ser maior que 71. 3. Prove que se N é ímpar então para qualquer bijeção

p : IN IN do conjunto IN = {1, 2, ... ,N} o produto P(p) = (1 – p(1))(2 – p(2)) ... (N – p(N)) é necessariamente par. (Dica: O produto de vários fatores é par se, e somente se, um dos fatores é par.)

Page 38: MA21 SOLUÇÕES DAS QUESTÕES DO LIVRO

Solução: Se N é ímpar então é da forma N = 2k + 1. Se P(p) fosse ímpar, todos os termos (i – p(i)) seriam ímpares para todo 1 i N. Sendo assim (N – p(N)) = (2k + 1 – p(2k + 1)) = 2k’ + 1 p(2k + 1) = 2(k – k’), o que implica que P(N) é par, logo P(p) não é ímpar. 4. Dado um conjunto de 25 pontos no plano tais que entre quaisquer 3 deles existe um par com distância menor que 1. Prove que existe um círculo de raio 1 que contém pelo menos 13 dos 25 pontos dados. Solução: (i) Inicialmente escolhemos dois pontos P e Q do plano cuja distância de P a Q é maior que 2, traçamos dois círculos C1 e C2 de raio 1 e centros P e Q respectivamente. Se todos os pontos estiverem no interior de um dos dois círculos o problema está resolvido. Caso contrário teríamos um dos pontos exterior aos dois círculos o que contraria o fato de que entre quaisquer 3 deles um par existe um par com distância menor que 1. (ii) Vamos analisar a situação os círculos C1 e C2 possuem intersecção é não vazia, ou seja, C1 e C2 possuem intersecção possui d pontos, com isso, cada círculo tem pelo menos 23 – d pontos. Assim,

cada região que não é intersecção entre os dois círculos tem na “pior” das hipóteses

com 0 ≤ d ≤ 23. Se d = 0, temos a situação (i), caso contrário, um dos círculos terá pelo

menos 13 pontos. . 5. Prove que entre quaisquer 5 pontos escolhidos dentro de um triângulo eqüilátero de lado 1 sempre existe um par destes cuja distância não é maior que 0,5. Solução: Dividindo o triângulo equilátero de lado 1 em quatro triângulos eqüiláteros de lado 1/2 como a figura que se segue.

Page 39: MA21 SOLUÇÕES DAS QUESTÕES DO LIVRO

Logo, pelo PCP pelo menos dois pontos devem estar no mesmo triângulo, uma vez que temos 4 triângulos e 5 pontos.

Assim, como a maior distância entre dois pontos que estão num triângulo equilátero ocorre quando estes estiverem no vértice do triângulo e como os 4 triângulos menores tem lados ½, então pelo menos dois de seus pontos estão a uma distância de no máximo ½. . 6. Marquemos todos os centros dos 64 quadrinhos de um tabuleiro de xadrez de 8x8. É possível cortar o tabuleiro com 13 linhas retas que não passem pelos pontos marcados e de forma tal que cada pedaço de recorte do tabuleiro tenha no máximo um ponto marcado? Solução: Temos marcados no tabuleiro 64 pontos, estando alinhados em 8 linhas e 8 colunas. Para separarmos os 8 pontos pertencentes as 8 colunas são necessárias 7 linhas e para dividirmos os 8 pontos pertencentes as 8 linhas são necessárias 7 linhas, totalizando 14 linhas. Caso sejam usadas 13 linhas poderemos dividir o tabuleiro em 56 espaços, como temos 64 pontos, pelo Princípio da Casa dos Pombos, teremos pelo menos 8 casas com 2 pontos. Logo, não é possível cortar o tabuleiro com 13 linhas retas que não passem pelos pontos marcados e de forma tal que cada pedaço de recorte do tabuleiro tenha no máximo um ponto marcado. 7. Prove que existem duas potências de 3 cuja diferença é divisível por 1997. Solução: Existem 1997 possíveis restos pela divisão por 1997. Considere a sequência das potências de 3: 30, 3¹, 3², 3³, ...,31997. Esta sequência é composta de 1998 números. Portanto, pelo PCP, dois desses, digamos 3n e 3m, com n > m, têm mesmo resto quando divididos por 1997. Logo, a sua diferença 3n– 3m é divisível por 1997. 8. São escolhidos 6 números quaisquer pertencentes ao conjunto

A = {1, 2, ...., 10}.

Prove que existem dois destes seis números cuja a soma é ímpar. Solução: Sabemos que a soma de dois números naturais é ímpar se, os números não tiverem a mesma paridade, isto é, se um for par e o outro for ímpar.

Page 40: MA21 SOLUÇÕES DAS QUESTÕES DO LIVRO

Como o conjunto A tem 5 números pares e 5 números ímpares e devemos escolher 6 números quaisquer, pelo PCP, teremos dentre os seis números escolhidos, pelo menos um número par e um número ímpar escolhido, onde a soma é ímpar. 9. Seja x um número real arbitrário. Prove que entre os números

x, 2x, 3x, ..., 101x existe um tal que sua diferença com certo número inteiro é menor 0,011. Solução:

Seja x um número real tal que, para certo inteiro se tenha a.x [, +1], para a {1, 2, ...,

101} e x .

Considere os conjuntos I = {x0 = 1, x1, x2, ..., x99, x100 = +1} com x1< x2< ...< x99 <

x100, e xi – xi – 1 = 0,01, i {1, 2, ..., 100}.

Considere os intervalos Ii = [xi – 1, xi] e x = ou 101.x = + 1, nada há a demonstrar.

Suponha que x ≠ e 101.x ≠ + 1.

Temos # { Ii } = 100 e # { x, 2x, 3x, ..., 101x} = 101. Logo, temos 101 elementos para acomodar em 100 intervalos de comprimento 0,01. Segue que teremos dois números em um único intervalo. Se x > 0, serão 100x e 101x. Se x < 0, serão x e 2x. Em qualquer caso teremos dois números num mesmo

intervalo, no qual um dos extremos é inteiro. Logo devemos ter x – < 0,01 < 0,011 ou + 1 – 101x

< 0,01 < 0,011. 10. Mostre que entre nove números que não possuem divisores primos maiores que cinco, existem dois cujo produto é um quadrado. Solução: Seja A= {x1, x2, x3, x4, x5, x6, x7, x8, x9} o conjunto de nove números que não possuem divisores primos maiores que cinco. Os números que não possuem divisores primos maiores que cinco são da forma xI

= 2.3.5.

Para que o produto de dois números xi.xj seja um quadrado, devemos ter xI = 2.3.5 e xj = 2’.3’.5’,

com i,j {1, 2, 3, 4, 5, 6, 7, 8, 9}, ou seja, xi.xj = (2.3.5 ).( 2’.3’.5’ ) = 2+’.3+’.5+’, onde + ’,

+ ’ e + ’ necessariamente devem ser pares, temos as seguintes possibilidades:

Page 41: MA21 SOLUÇÕES DAS QUESTÕES DO LIVRO

Possibilidades e ’ e ’ e ’ + ’, + ’, + ’

1ª Pares Pares Pares Pares

2ª Pares Pares Ímpares Pares

3ª Pares Ímpares Pares Pares

4ª Pares Ímpares Ímpares Pares

5ª Ímpares Pares Pares Pares

6ª Ímpares Ímpares Ímpares Pares

7ª Ímpares Pares Ímpares Pares

8ª Ímpares Ímpares Ímpares Pares

Então existem 8 possibilidades para que o produto seja quadrado. Pelo P.C.P., as 8 possibilidades são as casas e os 9 elementos de A são os pombos, e consequentemente, dentre os 9 elementos de A, o produto de pelo menos dois deles é um quadrado. 11. Um disco fechado de raio um contém sete pontos, cujas distâncias entre quaisquer dois deles é maior ou igual a um. Prove queo centro do disco é um destes pontos. Solução: 12. Na região delimitada por um retângulo de largura quatro e altura três são marcados seis pontos.

Prove que existe ao menos um par destes pontos cuja distância entre eles não é maior que √ . Solução: Dividimos o retângulo em quadrados de lado unitário, como mostra a figura abaixo. Posicionamos 4 destes pontos nos vértices do retângulo maior. Em seguida posicionamos o quinto ponto no centro do

retângulo maior (onde do centro aos outros teremos 2,5 > √ ). Ao acrescentarmos o sexto ponto, em qualquer região, ela não será maior que 2 em relação a 2 pontos mais próximos (lembrando que 2 <

√ ).

13. Seja a um número irracional. Prove que existem infinitos números racionais r = p/q tais que |a – r| j< 1/q2. Solução: 14. Suponha que cada ponto do reticulado plano é pintado de vermelho ou azul. Mostre que existe algum retângulo com vértices no reticulado e todos da mesma cor. Solução: Se imaginarmos 3 linhas horizontais, as quais possuam 3 ponto de cores alternadas (vermelho ou azul) e alinhados sobre uma reta perpendicular às horizontais; para cada paralela ao alinhamento desses pontos teremos 2x2x2 = 8 possibilidades de disposição de cores aos pontos de intersecção das paralelas com as horizontais. Então, sabemos que após 8 retas paralelas irá repetir algum padrão das 8 possibilidades, e iremos ter o possível retângulo.

Page 42: MA21 SOLUÇÕES DAS QUESTÕES DO LIVRO

15. Um certo livreiro vende pelo menos um livro por dia. Sabendo que o livreiro vendeu 463 livros durante 305 dias consecutivos, mostre que em algum período de dias consecutivos o livreiro vendeu exatamente 144 livros. Solução:

A soma de livros vendidos do 1º ao último dia é 463. Considere Sn como a soma dos livros vendidos do

1º dia ao dia n, com 1 305. Temos que:

1 S1 S2 <S3 <S4 < S305 = 463.

Note S4- S1 é a quantidade de livros vendidos entre o 1º e o 4º dia. Considere agora 1 p Devemos mostrar que existe p e q tal que Sp- Sq= 144.

Observe que a seqüencia abaixo é crescente, pois é vendido por dia pelo menos 1 livro.

S1 S2 < S3 < S4 < S305 (I), logo

144+S1 144+S2 <144+S3 <144+S4 < S305 (II)

Das sequencias (I) e (II) temos:

305+305= 610 números e

463+144= 607 valores possíveis.

Pelo PCB, os números equivalem aos pombos e os possíveis valores as casas, logo Sp seqüencia (I) e Sq+ 144 à seqüencia (II).

Portanto em algum período de dias consecutivos o livreiro vendeu exatamente 144 livros.

Page 43: MA21 SOLUÇÕES DAS QUESTÕES DO LIVRO

Capítulo 5 – Soluções dos Exercícios

1. De quantas maneiras podemos escolher três números distintos do conjunto I50 = {1, 2, 3, ..., 49, 50 } de modo que sua soma seja

a) um múltiplo de 3?

Devemos escolher números cujas classes de restos são do tipo 0 – 0 – 0 ou 1 – 1 – 1 ou 0 – 1 – 2 ou 2 – 2 – 2.

Classe de resto 0 são 16 números, de resto 1 ou 2 são 17 números cada.

Para escolher três números de resto 0 num total de 16 temos 5603

16

maneiras.

Para escolher três números de resto 1 ou 2 num total de 17 temos 1360680.23

17.2

maneiras.

Para escolher um número de resto 0, um número de resto 1 e um número de resto 2 temos, pelo principio multiplicativo, 16 . 17 . 17 = 4624 maneiras.

Logo teremos 560 + 1360 + 4624 = 6544 maneiras distintas de escolher os três números cuja soma será múltiplo de três.

b) um número par?

Podemos escolher três números pares num total de 25, ou seja, 23003

25

maneiras.

Podemos escolher dois números ímpares num total de 25, ou seja, 3002

25

maneiras e um número

par num total de 25. Portanto teremos 300 . 25 = 7500 maneiras.

Finalmente pelo principio aditivo teremos um total de 2300 + 7500 = 9800 maneiras distintas de escolher os três números cuja soma será par.

Page 44: MA21 SOLUÇÕES DAS QUESTÕES DO LIVRO

2. Considere o conjunto In = {1, 2, 3, ..., n – 1, n}. Diga de quantos modos é possível formar subconjuntos de k elementos nos quais não haja números consecutivos?

Considerando um caso particular se o conjunto fosse {1, 2, 3, 4, 5, 6}, teríamos 4 opções para formarmos 3 subconjuntos onde não existiriam números consecutivos. Seriam os seguintes subconjuntos: {1, 3, 5} {1, 3, 6} {1, 4, 6} {2, 4, 6}

É lógico que este processo de enumeração é exaustivo e nada prático, então, vamos tentar generalizar.

Usaremos o símbolo + para os elementos que farão parte do subconjunto e o símbolo - para os que não farão parte dele.

Para o exemplo dado, com um conjunto de 6 elementos e subconjuntos de 3 elementos, teríamos 3 símbolos + e 3 símbolos - que, em cada subconjunto não poderiam estar seguidos.

Para o subconjunto {1, 3, 5}, a simbologia seria: + - + - + -.

Devemos perceber que, para 6 elementos, ficam definidos 7 posições possíveis (n + 1), conforme figura abaixo.

- - -

Fixando os 3 lugares que seriam preenchidos pelos elementos que não farão parte do subconjunto, sobrariam 4 posições (n – k +1) para serem escolhidas 3 para serem preenchidas pelos que farão parte do subconjunto.

Note que, se temos 3 elementos que não vão participar do subconjunto, temos 3 + 1 (n – k +1) posições para serem ocupadas pelos outros 3 elementos, que farão parte do subconjunto. Logo, em nosso exemplo, temos uma única posição para os não participantes (-) e C4,3 para os participantes (+) do subconjunto.

Então, generalizando, teremos para n elementos kknC ,1 .

3. Considere as letras da palavra PERMUTA. Quantos anagramas de 4 letras podem ser formados, onde:

a) não há restrições quanto ao número de consoantes ou vogais?

b) o anagrama começa e termina por vogal?

c) a letra R aparece?

d) a letra T aparece e o anagrama termina por vogal?

a) 7 . 6 . 5 . 4 = 840 anagramas b) 3 . 5 . 4 . 2 = 120 anagramas

c) São 4 posições para colocar a letra R. Depois temos 6 . 5 . 4 = 120 maneiras de escolher as outras três letras. Portanto temos 4 . 120 = 480 anagramas com a letra R.

Page 45: MA21 SOLUÇÕES DAS QUESTÕES DO LIVRO

d) São três posições para colocar a letra T. Temos 3 . (5 . 4 . 3) = 180 anagramas com a letra T que terminam em vogal.

4. Calcular a soma de todos os números de 5 algarismos distintos formados com os algarismos 1; 3; 5; 7 e 9.

Teremos um total de 120 números todos da forma edcba 10.10.10.10. 234 . Sendo que a, b, c,

d, e podem assumir os valores 1, 3, 5, 7 e 9.

Somando esses algarismos teremos 1 + 3 + 5 + 7 + 9 = 25 (soma dos valores absolutos que corresponde à soma quando cada um deles estiver em uma ordem fixa). Como são 24 números com os algarismos ocupando uma posição fixa teremos 25 x 24 = 600.

Para somar todos os números devemos usar a decomposição do total que será dada por

)(10).(10.)(10).(10).( 234 edcbaedcbaedcbaedcbaedcba

multiplicado por 24. Como (a + b + c+ d + e) x 24 = 600, temos por soma o produto

110101010.600 1234 = 600 x 11111 = 6666600

5. Quantos números podem ser formados pela multiplicação de alguns ou de todos os números 2; 2; 3; 3; 3; 5; 5; 6; 8; 9; 9?

Tais números são do tipo cba 5.3.2 , sendo 0 a 6; 0 b 8 e 0 c 2. Como são 7, 9 e 3

possibilidades para a, b e c, respectivamente, então são 7 . 9 . 3 = 189 possibilidades para cba 5.3.2 .

Mas 1, 2, 3 , 5, 8, 38, 38 . 5, 38 . 52 , 26 , 26 . 5 e 26 . 52 não podem ser obtidos como pede o texto. Assim, são 178 desses números.

6. Entre todos os números de sete dígitos, diga quantos possuem exatamente três dígitos 9 e os quatro dígitos restantes todos diferentes?

Temos um total de 4,9

3

7 .CP números de sete dígitos com três repetidos, veja figura abaixo.

9 9 9

3

7P 4,9C

Mas, devemos excluir os números iniciados em zero, que são em um total de 3,8

3

6 .CP

Então teremos 4,9

3

7 .CP - 3,8

3

6 .CP = 126 x 840 – 56 x 120 = 99120 números.

Page 46: MA21 SOLUÇÕES DAS QUESTÕES DO LIVRO

7. De quantas maneiras podemos distribuir 22 livros diferentes entre 5 alunos se 2 deles recebem 5 livros cada e os outros 3 recebem 4 livros cada?

Primeiro escolhemos os alunos de 3,32,5 . CC maneiras diferentes.

Depois escolhemos os livros de 4,44,84,125,175,22 .... CCCCC maneiras diferentes.

O total, pelo principio multiplicativo, será o produto das duas escolhas.

Teremos 3,32,5 . CC . 4,44,84,125,175,22 .... CCCCC = !2.!3.!4.!5

!223

= 56463835428000

8. Quantos são os números naturais de sete dígitos nos quais o dígito 4 figura exatamente 3 vezes e o dígito 8 figura exatamente 2 vezes?

Os números iniciados em 4 são iguais a 8.8...1 2,42,6 CC = 5760 ( 2,6C representa as maneiras de

acrescentar os outros dois dígitos 4 e 2,4C representa as maneiras de acrescentar os dois dígitos 8).

Os números iniciados em 8 são iguais a 8.8...1 3,51,6 CC = 3840 ( 1,6C representa as maneiras de

acrescentar o outro dígito 8 e 3,5C representa as maneiras de acrescentar os três dígitos 4).

Os números iniciados em outros dígitos, exceto o zero, são iguais a 8...7 2,33,6 CC = 3360 ( 3,6C

representa as maneiras de acrescentar os três dígitos 4 e 2,3C representa as maneiras de acrescentar

os dois dígitos 8).

Portanto, pelo princípio aditivo, temos a soma 5760 + 3840 + 3360 = 12960 números.

9. De quantas maneiras uma comissão de 4 pessoas pode ser formada, de um grupo de 6 homens e 6 mulheres, se a mesma é composta de um número maior de homens do que de mulheres?

Se o número de homens é maior que o número de mulheres então devemos ter 3 homens e 1 mulher ou 4 homens e 0 mulheres.

Para 3 homens e 1 mulher temos 120. 1,63,6 CC maneiras de escolher as comissões.

Para 4 homens e 0 mulheres temos 154,6 C maneiras de escolher as comissões.

Pelo principio aditivo temos 120 + 15 = 135 maneiras de escolher as comissões.

Page 47: MA21 SOLUÇÕES DAS QUESTÕES DO LIVRO

10. O comprimento de uma palavra é a quantidade de caracteres que ela possui. Encontre a quantidade de palavras de comprimento 5 que podemos formar fazendo uso de 10 caracteres distintos, de forma que não existam três caracteres consecutivos idênticos em cada palavra.

Usando os 10 caracteres temos 510 maneiras de formar as palavras de comprimento 5.

São indesejáveis as palavras cujo formato está mostrada na figura abaixo.

A A A B C

Grupo único

O grupo que repete pode ser contado como 10 . 3! (são 10 caracteres diferentes e temos três itens para permutar AAA, B e C.

Para as duas posições que sobram podemos ter 9 . 8 = 72 maneiras de escolha (pois esses caracteres não podem ser iguais).

Portanto temos 510 – 8.9.!3.10 = 95680 palavras.

11. Quantos números inteiros existem entre 1 e 10.000 que não são divisíveis por 3; 5 e 7?

Obs.: Fiz o exercício sem considerar que fossem múltiplos simultâneos.

São 3333 múltiplos de 3, 1999 múltiplos de 5 e 1428 múltiplos de 7.

São 666 múltiplos de [3, 5] = 15; 476 múltiplos de [3, 7] = 21 e 285 múltiplos de [5, 7] = 35.

São 95 múltiplos de [3, 5, 7] = 105.

Pelo princípio da inclusão e exclusão temos 10000 – [3333 + 1999 + 1428 – (666 + 476 + 285) + 105] = 4562 números que não são múltiplos de 3, 5 e 7.

Page 48: MA21 SOLUÇÕES DAS QUESTÕES DO LIVRO

12. Quantas são as permutações da palavra PROPOR nas quais não existem letras consecutivas iguais?

O método usado foi através da fixação de uma das letras e determinando as possibilidades de posicionar as demais letras sem haver repetição.

Cada linha da tabela abaixo mostra as posições escolhidas para uma letra.

Fixando o P dessa forma temos 2 . 1 . 1 . 1 = 2 possibilidades

Fixando o P dessa forma temos 2 . 1 . 2 . 1 = 4 possibilidades

Fixando o P dessa forma temos 2 . 1 . 1 . 1 = 2 possibilidades

Fixando o P dessa forma temos 2 . 1 . 1 . 1 = 2 possibilidades

Temos um total de 2 + 4 + 2 + 2 = 10 possibilidades usando a letra P.

Como são três letras, então temos 3 x 10 = 30 palavras nas quais não existem letras consecutivas iguais.

13. De quantos modos 6 casais podem sentar-se ao redor de uma mesa circular de tal forma que marido e mulher não fiquem juntos?

Primeiro posicionamos o grupo de mulheres que pode ser feito de 5! = 120 maneiras.

Em seguida devemos posicionar os homens observando a restrição imposta.

Designando os casais como (M1, H1); ... ; (M6, H6) sabemos que o Hi não pode ficar ao lado de Mi.

Na figura abaixo verificamos que existem duas posições proibidas para cada Hi.

Então podemos posicionar os homens de 4! = 24 maneiras distintas.

Logo, pelo principio multiplicativo, temos 24 x 120 = 2880 maneiras de posicionar os seis casais obedecendo à restrição imposta.

P 2 P 1 1 1

P 2 1 P 2 1

P 2 1 1 P 1

P 2 1 1 1 P

Proibido para H1

Proibido para H1

Page 49: MA21 SOLUÇÕES DAS QUESTÕES DO LIVRO

B R

4!

5! 5!

4! 3! 4!

5!

L

14. Quantas são as permutações das letras da palavra BRASIL em que o B ocupa o primeiro lugar, ou o R ocupa o segundo lugar, ou o L o sexto lugar?

No diagrama ao lado temos:

Anagramas em que o B ocupa o primeiro lugar 5!.

Anagramas em que o R ocupa o segundo lugar 5!.

Anagramas em que o L ocupa o sexto lugar 5!.

Anagramas contados com repetição de B e R 4!.

Anagramas contados com repetição de B e L 4!.

Anagramas contados com repetição de L e R 4!.

Anagramas com as três condições do problema 3!.

Portanto, pelo princípio da inclusão e exclusão, temos um total de 5! + 5! + 5! – (4! + 4! + 4!) + 3! = 294 anagramas que contemplam o enunciado.

15. De quantas formas podemos representar o número 15 como soma de vários números naturais?

Não encontramos um padrão de combinatória para a solução.

Foi resolvido, em sala, relacionando caso a caso e encontrou-se 176 formas.

16. Quantos quadrados perfeitos existem entre 40.000 e 640.000 que são múltiplos simultaneamente de 3, 4 e 5?

Os múltiplos simultâneos de 3, 4 e 5 são múltiplos de [3, 4 , 5] = 60 = 22 . 3 . 5. Seja M um número

natural múltiplo de 60 e que seja quadrado perfeito temos então que M = 60.Q = Q.5.3.22, onde

NQ de forma que tenhamos, no mínimo, Q = 27.5.3 e no máximo Q =

22 13.5.3.2 , pois pela

condição do problema M deve ser maior que 40000 e 60 . 27.5.3 = 22 7.5.3.5.3.2 = 2222 7.5.3.2 = 44100

e 60 . 22 13.5.3.2 =

222 13.5.3.2.5.3.2 = 608400 que representam, respectivamente, o primeiro

quadrado perfeito múltiplo de 60 maior que 40000 e o maior quadrado perfeito múltiplo de 60 menor que 640000.

Page 50: MA21 SOLUÇÕES DAS QUESTÕES DO LIVRO

Fazendo uma análise dos números quadrados perfeitos que cumprem o enunciado percebemos que

eles são da forma 900.2P , como 900.7900.4944100 2 e 900.26900.676608400 2 então

temos que 267 P . Portanto são 26 – 7 + 1 = 20 números quadrados perfeitos que cumprem as

condições do enunciado.

17. Oito amigos vão ao cinema assistir a um filme que custa um real. Quatro deles possuem uma nota de um real e quatro possuem uma nota de dois reais. Sabendo-se que o caixa do cinema não possui nenhum dinheiro, como eles podem organizar uma fila para pagar o filme permitindo o troco pelo caixa?

De acordo com a árvore de possibilidades acima temos 14 caminhos diferentes para o posicionamento dos oito amigos.

O grupo que possui um real cada pode ser posicionado de 4! = 24 maneiras na fila, da mesma forma os que possuem dois reais cada também podem ser posicionados de 4! = 24 maneiras na fila.

Então, pelo principio multiplicativo, temos um total de 14 . 24. 24 = 8064 maneiras de dispor os amigos na fila.

18. Se considerarmos todas as configurações do tabuleiro com duas torres que não se atacam, como no Exemplo 5.2, sem distinguir as torres, quantas configurações obteremos?

No exemplo 5.2 as torres são diferentes e representam 31367.64 2 maneiras distintas para

posicioná-las de forma que não se ataquem. Para o caso proposto as torres são idênticas portanto

temos 1568!2

7.64 2

configurações.

Page 51: MA21 SOLUÇÕES DAS QUESTÕES DO LIVRO

19. Continuando o problema anterior, generalize-o para 3, 4, 5, ... torres que não se atacam, encontrando também o número máximo de torres que podem ser colocadas no tabuleiro de modo que

duas delas não se ataquem.

Observando a solução do problema 18 e a figura ao lado temos que 64 representa o número de casas para colocar a primeira torre, 72 representa o número de casas que a segunda torre pode ocupar (posicionando a primeira sobra uma quantidade de casas equivalente à um quadrado com 7

casas de lado) e o !2 corrige o fato das torres serem iguais

(a permutação não gera uma nova disposição).

Para três torres temos !3

6.7.8 222

configurações, para

quatro torres temos !4

5.6.7.8 2222

e para n torres temos

!

18...7.8222

n

n

O máximo acontece para n = 8. Basta posicionar todas as torres na diagonal.

20. Tente fazer o problema anterior para cavalos de xadrez.

Iniciei com as figuras abaixo, mas não consegui concluir.

Cavalo nos vértices.

Cavalo 1 4 opções

Cavalo 2 61 opções

Cavalo na segunda ou sétima coluna das laterais.

Cavalo 1 8 opções

Cavalo 2 60 opções

Cavalo na 3a ou 4a ou 5a ou 6a coluna das laterais.

Cavalo 1 16 opções

Cavalo 2 59 opções

Page 52: MA21 SOLUÇÕES DAS QUESTÕES DO LIVRO

21. Mostre que em toda sequência de n2 +1 inteiros distintos possui uma subsequência crescente de n + 1 elementos ou uma subsequência decrescente de n + 1 elementos.

Seja um inteiro positivo e suponhamos, por contradição, que haja uma sequência de 2+1 inteiros

distintos que não contenha uma subsequência monótona (cresceste ou decrescente) de tamanho +1, ou seja, todas as subsequências monótonas de tem tamanho , no máximo. Seja um elemento da

sequência . Associamos a um par ordenado de inteiros , , onde é o comprimento da maior subsequência crescente de que começa em e é o comprimento da maior subsequência

decrescente de que começa em . Sobre as subsequências monótonas de podemos afirmar que:

𝑖) o máximo de pares ordenados de inteiros utilizados é 2. De fato, como o comprimento de uma subsequência monótona de é no máximo igual a , utilizamos inteiros de 1 a . Logo, utilizamos no

máximo . = 2 pares ordenados de inteiros; 𝑖𝑖) dois elementos distintos de não podem estar associados ao mesmo par ordenado de inteiros. De

fato, suponhamos que e são elementos de S, com ≠ . Seus pares ordenados são , e , e como ≠ , temos que < ou > . Se < , afirmamos que > ·, pois sabemos que existe uma

subsequência crescente de comprimento começando em . Se inserirmos no começo dessa subsequência, obtemos uma subsequência crescente de comprimento +1. Assim, ≥ +1 ou,

equivalentemente, > . Assim e têm pares ordenados diferentes. Da mesma forma, se > , então > e, novamente e estão associados a pares ordenados diferentes.

Entretanto, essas duas afirmações levam a uma contradição. Há apenas 2 pares ordenados de inteiros

diferentes, e tem 2+1 elementos. Pelo PCP (Principio da Casa dos Pombos), dois dos elementos devem ter o mesmo par ordenado. Entretanto, isto contradiz o fato de que dois elementos quaisquer

não podem estar associados ao mesmo par ordenado. Portanto deve ter uma subsequência monótona de comprimento +1.

22. Encontre o número de zeros que termina o número 2010!.

Usando a técnica de contar os grupos das potências de 5 (que multiplicados pelas potências de 2 resulta em zeros) temos:

Temos 2010 : 5 = 402 grupos de 51.

Temos 402 : 5 = 80 grupos de 52.

Temos 80 : 5 = 16 grupos de 53.

Temos 16 : 5 = 3 grupos de 54.

Como 54 é a maior potência de 5 presente em 2010, temos um total de 402 + 80 + 16 + 3 = 503 zeros na terminação de 2010!.

Page 53: MA21 SOLUÇÕES DAS QUESTÕES DO LIVRO

23. O jogo do 7 consiste em lançar dois dados e somar o número obtido nas suas faces. Caso a soma seja 7, o jogador A ganha dois reais do jogador B. Caso a soma não seja 7, o jogador B ganha um real de A. Pergunta-se: quem leva vantagem?

Quem leva vantagem é o jogador B, pois no lançamento de dois dados temos 3662 somas

diferentes. Dentre estas apenas 6 dão total sete. Portanto em cada jogada temos uma probabilidade de

6

1 a favor do jogador A e

6

5 a favor do jogador B.

24. A função de Euler associa a cada número natural n o valor (n) igual ao número de inteiros

positivos menores ou iguais a n relativamente primos com n. Ou seja, 1,;1)( nmnmn

Usando os princípios estudados, mostre que se n se decompõe

em fatores primos como k

kpppn

.... 21

21 , então

kpppnn

11...

11.

11)(

21

.

k

k

k p

p

p

p

p

pn

pppnn

1...

1.

1.

11...

11.

11)(

2

2

1

1

21

k

k

ppp

pppnn

....

1...1.1.)(

21

21 , substituindo k

kpppn

.... 21

21 temos:

k

k

kppp

ppppppn k

....

1...1.1.....)(

21

21

2121

dividindo as potências dos fatores primos de n resulta

1...1.1.....)( 21

11

2

1

121

kk ppppppn k

Page 54: MA21 SOLUÇÕES DAS QUESTÕES DO LIVRO

Capítulo 6 - Soluções dos Exercícios QUESTÃO 1

Se qn denota a soma qn = 12 + 22 + ... + n2, prove que para todo n N 6

)121

.n)(n(nqn

6

)32)(2)(1()1(...21

Logo,

6

)32)(2)(1(

6

)2

3)(2(2)1(

6

)672)(1(

6

)1(6)12()1(

6

)1(6221

6

)1(6121)1(21

)1(6

121)1(21

: teremos6

22121 de membros dois nos )1( Somando

:ãoDemonstraç

6

)32)(2)(1()1(...21

:então ,1para indução, de Tese

6

)22)(1(...21

:então , com para a verdadeiré que indução, de Hipótese- 2

16

)112111 11

:pois ,1para a verdadeiré afirmação A - 1

finita. indução doprovar Vamos

2222

22

22222

22222

2222

2222

222

2

kkkkk

kkkkkk

kkkkkkkk)k)(k(k

k)k)(k(kkk...

k)k)(k(k

kk...

)k)(k(kk...k

kkkkk

kn

kkkk

Nkkn

.)(( qq

n

nn

QUESTÃO 2 Use o princípio da indução para provar as seguintes afirmações:

(a) 32n+1 + 2n+2 é divisível por 7 para todo n N;

7.por divisível é Que

)23.3(73.6143.27)4

3.37(83.2723

: teremoshipótesea dosubstituin ,2.83.2723

teremosTese,a ndoDesenvolve

:ãoDemonstraç

7por divisível é 23

:então ,1para indução, de Tese

4

372 logo ,723 então 7,por divisível é 23

:então , com para a verdadeiré que indução, de Hipótese- 2

7.por divisível é que 352323

:pois ,1para a verdadeiré afirmação A - 1

finita. indução doprovar Vamos

2222

2332

2332

332

12212212

332111.2

qqq

kn

qq

Nkkn

n

kkkk

kkk

kkkk

kk

kkkkkk

Page 55: MA21 SOLUÇÕES DAS QUESTÕES DO LIVRO

(b) a soma dos cubos de três números naturais consecutivos é divisível por 9; Demonstrar que n2 + (n+1)2 + (n+2)2 é divisível por 9.

9.por divisível é Que

)993(9)3()2()1(

272799)3()2()1(

)3()1(9)1()3()2()1(

:que concluimos na tese, )2( dosubstituin

)1(9)2(

teremos,hipótesena )2( Isolando

:ãoDemonstraç

9por divisível é )3()2()1(

:então ,1para indução, de Tese

9)2()1( então 9,por divisível é )2()1(

:então , com para a verdadeiré que indução, de Hipótese- 2

9.por divisível é 36321

:pois ,1para a verdadeiré afirmação A - 1

finita. indução doprovar Vamos

2333

233333

3333333

3

333

3

333

333333

333

nnqnnn

nnnnqnnn

nnnqnnnn

n

nnqn

n

nnn

kn

qnnnnnn

Nkkn

n

1 1

Vamos provar do indução finita.

1 - A afirmação é verdadeira para 1, pois:

(10 9.1 10) 817 7. 7. 7

81 81

2 - Hipótese de indução, que é verdadeira para com , então:

7 77 777 ... 777...7k vezes

n

n k k N

1

2 2

1

1

(10 9 10)7.

81

Tese de indução, para 1, então:

(10 9( 1) 10) (10 9 19)7 77 777 ... 777...7 7. 7.

81 81

Demonstração:

Somando 777...7 nos dois membros da hipótese tere

k

k k

k vezes

k vezes

k

n k

k k

1

1 1

1 1

1 1

1

mos

(10 9 10)7 77 777 ... 777...7 777...7 7. 777...7

81

(10 9 10) (10 9 10)7. 777...7 7. 7.111...1

81 81

7.111...1 7.(10 ...

k

k vezes k vezes k vezes

k k

k vezes k vezes

k

k vezes

k

k k

n 1

2 2

1 1 12

1 12

1

x 110 10 1), lembrando que ... 1,então

1

10 1 10 1 9 9.10 97.(10 ... 10 10 1) 7. 7. . 7.

10 1 10 1 9 81

(10 9 10) (10 9 10)7. 7.111...1 7. 7.(10 ... 10 10 1)

81 81

n

k k kk

k kk

k vezes

x x xx

k k

1 1 1 2(10 9 10) 9.10 9 (10.10 9 10 9) (10 9 19)7. 7. 7. 7.

81 81 81 81

k k k kk k k

Page 56: MA21 SOLUÇÕES DAS QUESTÕES DO LIVRO

1

Vamos provar do indução finita.

1 - A afirmação é verdadeira para 1, pois:

( 1)( 2)( 3)...( ) 2 .1.3.5...(2 1)

(1 1) 2 2 .1.(2.1 1) 2.1 2

2 - Hipótese de indução, que consideraremos verdadeira par

n

n

n n n n n n

1

1

a com , então:

( 1)( 2)( 3)...( ) 2 .1.3.5...(2 1)

Tese de indução, para 1, então:

( 1 1)( 1 2)( 1 3)...( 1 1) 2 .1.3.5...2( 1)

( 2)( 3)( 4)...( )(2 1)(2 2) 2 .1.3.5...(

k

k

k

n k k N

k k k k k k

n k

k k k k k k

k k k k k k k

hipótese de indução

2 1)

Demonstração:

Vamos trabalhar com o primeiro mebro da tese

( 2)( 3)( 4)...( )(2 1)(2 2)

( 2)( 3)( 4)...( )(2 1).2( 1) ( 1)( 2)( 3)( 4)...( ).2(

k

k k k k k k k

k k k k k k k k k k k k k

1

2 1)

Substituindo a hipótese teremos :

2 .1.3.5...(2 1).2(2 1) 2 .1.3.5...(2 1).(2 1) . Temos a validade de nossa tese.k k

k

k k k k

QUESTÃO 3 Use o princípio da indução para provar as seguintes desigualdades:

A desigualdade nnnn baba 12 não é válida para todo n natural, pois ela não é válida para

1n . Para 1n , ela fica baba , que não é verdadeiro. Mas, de fato, a desigualdade

nnnn baba 12 é válida para todo 2n . Ela é válida para 2n , pois para 2n , ela fica

2222 baba , que equivale a 22222 222 abbaba , que equivale a 0222 abba ,

que equivale a 02ba , que é verdadeiro, já que ba . Suponha que a igualdade vale para n e

vamos mostrar que ela vale para 1n , isto é, vamos mostrar que 1112

nnnn baba , isto é,

vamos mostrar que 1112

nnnnbaba . De fato, como bababa

nn

1, 0ba e,

por hipótese de indução, nnnnbaba 12 , então bababa nnnn

112 . Assim, para

concluir que 1112

nnnnbaba é suficiente mostrar que 111 22 nnnnnn bababa .

Mas, 111 22 nnnnnn bababa equivale a bababa nnnn 112 , que equivale a

nnnnnn abbababa 1111 22 , que equivale a nnnn abbaba 11, que equivale a

nnnn abbabbaa , que equivale a 0 baba nn, que é verdadeiro, pois se ba , então

0 nn ba e 0ba , sendo portanto 0 baba nn, e se ba , então 0 nn ba e 0ba ,

sendo portanto 0 baba nn.

Page 57: MA21 SOLUÇÕES DAS QUESTÕES DO LIVRO

A desigualdade nn

1

2

1

1

1 não é válida para todo n natural, pois ela não é válida para

1n . Para 1n , ela fica 11

1 , que não é verdadeiro. Mas, de fato, a desigualdade

nn

1

2

1

1

1 é válida para todo 2n . Ela é válida para 2n , pois para 2n , ela fica

22

1

1

1 , que equivale a 2

2

12

, que equivale a 212 , que é verdadeiro. Suponha

que a igualdade vale para n e vamos mostrar que ela vale para 1n , isto é, vamos mostrar que

11

11

2

1

1

1

n

nn . De fato, como, por hipótese de indução,

nn

1

2

1

1

1 , então

1

1

1

11

2

1

1

1

nn

nn . Assim, para concluir que

11

11

2

1

1

1

n

nn é suficiente mostrar que 1

1

1

n

nn . Mas,

11

1

n

nn equivale a

1

1

11

n

n

nn, que equivale a 111 nnn , que

equivale a nnn 1 , que equivale a 21 nnn , que equivale a 0n , o que é verdadeiro, pois

2n .

A desigualdade 24

13

2

1

2

1

1

1

nnn não é válida para todo n natural, pois ela não é válida para

1n . Para 1n , ela fica 23

13

2

1 , que não é verdadeiro. Mas, de fato, a desigualdade

24

13

2

1

2

1

1

1

nnn é válida para todo 2n . Ela é válida para 2n , pois para 2n , ela fica

24

13

4

1

3

1 , que equivale a

24

13

12

7 , que é verdadeiro. Suponha que a igualdade vale para n e vamos

mostrar que ela vale para 1n , isto é, vamos mostrar que

24

13

22

1

12

1

2

1

3

1

2

1

nnnnn . De fato, como, por hipótese de indução,

24

13

2

1

2

1

1

1

nnn , então

1

1

24

13

2

1

2

1

nnn . Como

1

1

24

13

2

1

2

1

nnn ,

então 22

1

12

1

1

1

24

13

22

1

12

1

2

1

3

1

2

1

nnnnnnnn . Assim, para concluir que

24

13

22

1

12

1

2

1

3

1

2

1

nnnnn é suficiente mostrar que

24

13

22

1

12

1

1

1

24

13

nnn. Mas,

24

13

22

1

12

1

1

1

24

13

nnn equivale a

22

1

12

1

1

1

nnn, que equivale a

2212

34

1

1

nn

n

n, que equivale a

Page 58: MA21 SOLUÇÕES DAS QUESTÕES DO LIVRO

3412212 nnnn , que equivale a 374264 22 nnnn , que equivale a 01n , o

que é verdadeiro. QUESTÃO 4 Mostre a seguinte identidade trigonométrica

2

( 1)cos cos( 1) 1cos 2cos 2 .... cos

24

n nx n n xx x n nx

xsen

Vamos considerar a identidade 2

2(1 cos )2

4x

xsen , logo nossa identidade a ser demonstrada pode

ser reescrita com o

P(n): ( 1)cos cos( 1) 1

cos 2cos 2 .... cos2(1 cos )

n nx n n xx x n nx

x

Vamos demonstrar P(n) utilizando indução finita.

Vamos inicialmente testar n = 1, então

22cos 2 s(2) 1

(1) cos cos (V)2(1 cos ) 2(1 cos ) 2(1 cos )

2cos (1 cos )2cos 2cosx co xP x x

x x x

x xx x

Hipótese: Vamos supor que P(n) é verdadeira para n = k

( 1)cos cos( 1) 1( ) cos 2cos 2 .... cos

2(1 cos )

k kx k k xP k x x k kx

x

Tese: Vamos demonstrar que P(n) é verdadeira para n=k+1.

( 2)cos( 1) ( 1)cos( 2) 1( 1) cos 2cos 2 .... cos ( 1)cos( 1)

2(1 cos )

k k x k k xP k x x k kx k k x

x

Somando (k+1)cos(k+1)x aos dois membros da hipótese teremos:

( 1)cos cos( 1) 1cos 2cos 2 .... cos ( 1)cos( 1) ( 1)cos( 1)

2(1 cos )

( 1)cos cos( 1) 1 2( 1)cos( 1) 2cos ( 1)cos( 1)

2(1 cos )

cos( 1) 2 2 2( 1)cos ( 1)cos 1

2(1

k kx k k xx x k kx k k x k k x

x

k kx k k x k k x x k k x

x

k x k k k x k kx

( )

cos( 1) 2 2( 1)cos ( 1)cos 1

cos ) 2(1 cos )

( 2)cos( 1) ( 1) 2cos cos( 1) cos 1

2(1 cos )

( 2)cos( 1) ( 1)cos( 2) ( 1) (2cos cos( 1) cos cos( 2) ) 1

2(1 cos

I

k x k k x k kx

x x

k k x k x k x kx

x

k k x k k x k x k x kx k x

)x

Page 59: MA21 SOLUÇÕES DAS QUESTÕES DO LIVRO

Vamos provar que (I) = 0.

Primeiramente observe que:

cos(k+2)x=cos[(k+1)x+x]=cos(k+1)x.cosx-sen(k+1)x.senx (II)

e

coskx=cos[(k+1)x-x]=cos(k+1)x.cosx+sen(k+1)x.senx (III)

Então trabalhando com (I) teremos que

2cos cos( 1) cos cos( 2)

2cos cos( 1) (cos(k+1)x.cosx+sen(k+1)x.senx+cos(k+1)x.cosx-sen(k+1)x.senx)=

2cos cos( 1) 2cos cos( 1) 0

Com isso provamos que a identidade trigonométrica é verdad

x k x kx k x

x k x

x k x x k x

eira.

QUESTÃO 5 Um torneio de xadrez tem n jogadores. Cada jogador joga uma única partida com cada um dos outros jogadores. Calcule o número total de partidas realizadas no torneio.

O número de jogos realizados no torneiro será dado pela

2

n

n !

2 2! 2 !C

n

n

. Observe que são

n jogadores, a partida é realizada entre dois jogadores e cada dupla joga uma única vez. Vamos considerar P(n) como sendo o total de jogos realizados, ou seja que

P(n) =

!

2! 2 !

n

n . Vamos demonstrar a validade de P(n) para todo n natural maior que 1, utilizando

indução finita. Para n = 2 é fácil verificar que será uma única partida.

2!(2) 1

2! 2 2 !P

Como hipótese vamos considerar que P(n) é válida para n = k, com k natural e maior que 2.

! k.(k-1)( )

2! 2 ! 2

kP k

k

Tese: Vamos demonstrar que para n = k+1, P(n) também é verdadeira.

( 1)! (k+1).k( 1)

2! 1 ! 2

kP k

k

Para demonstrar a validade de P(k+1) devemos perceber inicialmente que se para k jogadores o

número de jogos é dado por .( 1)

( )2

k kP k

(hipótese de indução), ao aumentarmos um jogador ,

teremos k jogos a mais pois, esse novo jogador irá jogar com os k outros jogadores. Somando k à hipótese teremos:

2 2

( ).( 1) ( 1)2

2 2 2 2P k

k k k kk k k k kk

Temos então a validade de nossa tese.

Page 60: MA21 SOLUÇÕES DAS QUESTÕES DO LIVRO

QUESTÃO 6

2

3333

2

22223223

2

3

2

33333

2

2

3333

2

3333

2

2

3

2

)2)(1()1(...321

Logo.

2

)2)(1(

4

)2()1(

4

44)1(

4

)1(4)1()1(

2

)1(

)1(2

)1()1(...321

: temoshipóteseda membros dois nos 1 Somando

:ãoDemonstraç

2

)2)(1()1(...321

:então ,1para indução, de Tese

2

)1(...321

:então , com para a verdadeiré que indução, de Hipótese- 2

112

)11.(11

:pois ,1para a verdadeiré afirmação A - 1

finita. indução doprovar Vamos

kkk

kk

kkkkkkkkk

kk

kkk

kk

)(k

kkk

kn

kkk

Nkkn

n

QUESTÃO 7

32

13

2

11

2

111

)1(

11

1

1).1(1

)1(

11

1

11

: temosNewton de binômio do mentodesenvolvi Pelo

:ãoDemonstraç

31

11

:então ,1para indução, de Tese

31

131

1

:então , com para a verdadeiré que indução, de Hipótese- 2

321

11

:pois ,1para a verdadeiré afirmação A - 1

finita. indução doprovar Vamos

1

11

2

1

2

1

0

1

1

1

1

1

1

k

k

nk

k

nk

n

k

nn

k

n

n

n

n

n

kk

k

kn

k

kk

kn

k

na

na

kn

qkk

a

Nkkn

a

n

Page 61: MA21 SOLUÇÕES DAS QUESTÕES DO LIVRO

QUESTÃO 8

2

141...

:logo ,2

141

2

1421 Sendo,

2

1421...

2

141...

2

141...

:hipóteseda membros dois aosa somar vamosentão 0,a Sendo

:ãoDemonstraç

2

141...

:então ,1para indução, de Tese

2

141...

:então , com para a verdadeiré que indução, de Hipótese- 2

2

141

:pois ,1para a verdadeiré afirmação A - 1

finita. indução doprovar Vamos

1

1

1

1

aaaaaa

aaa

aaaaaaa

aaaaaaa

aaaaaaa

aaaaa

kn

aaaaa

Nkkn

aa

n

radicaisk

radicaisk

radicaisk

radicaisk

radicaisk

radicaisk

radicaisk

Page 62: MA21 SOLUÇÕES DAS QUESTÕES DO LIVRO

QUESTÃO 9

133.por divisível é Que

)111728(133)11.111728(1331211

11.11.1441728.13311.1112

11133.172811.13311211

: teremoshipótesea dosubstituin ,12.172811.13311211

teremosTese,a ndoDesenvolve

:ãoDemonstraç

133por divisível é 1211

:então ,1para indução, de Tese

12

1113321 logo ,1331211 então 133,por divisível é 1211

:então , com para a verdadeiré que indução, de Hipótese- 2

133.por divisível é que 133.2312111211

:pois ,1para a verdadeiré afirmação A - 1

finita. indução doprovar Vamos

22323

232

323

2323

323

22122122

3311.221

kkkk

kkk

kkk

kkkk

kk

kkkkkk

qq

qq

kn

qq

Nkkn

n

QUESTÃO 10

7.por divisível é Que

)23.3(73.6143.27)4

3.37(83.2723

: teremoshipótesea dosubstituin ,2.83.2723

teremosTese,a ndoDesenvolve

:ãoDemonstraç

7por divisível é 23

:então ,1para indução, de Tese

4

372 logo ,723 então 7,por divisível é 23

:então , com para a verdadeiré que indução, de Hipótese- 2

7.por divisível é que 352323

:pois ,1para a verdadeiré afirmação A - 1

finita. indução doprovar Vamos

2222

2332

2332

332

12212212

332111.2

qqq

kn

qq

Zkkn

n

kkkk

kkk

kkkk

kk

kkkkkk

Page 63: MA21 SOLUÇÕES DAS QUESTÕES DO LIVRO

Questão 11

2n+2+ 6 1Mostre que para todo n temos que é um múltiplo de 11.3Z 2

n

Vamos fazer a demonstração por indução finita.

Consideremos n = 0.

Substituindo teremos 2.0+2 26.0 1 1

= + = que é múltiplo de 11.3 32 2 11

Hipótese de Indução - Suponhamos que seja verdade para n=k , com k + Z , temos que

2.k+2 6. 1 + = com q 11q3 2 Z

k

Tese

Vamos demonstrar que para n = k+1 , 2.k+4 6. 7

11 divide 3 2k

Desenvolvendo nossa tese teremos 2.k+4 2k+2 26. 7 6 7

= . .3 3 32 2 2k k

.

Agora iremos trabalhar com a hipótese.

2.k+2 2.k+26. 1 6. 1 = ( )11q 11q3 32 2

k kI

Substituindo (I) na tese teremos que

2 6 66 7 6 1 6 7 6 66 1( ). . . . )9.11 9. 9.11 9.11 11(9 10.11 3 18.2 128 110.22 2 2 2 2 2 22k kk k k k kk q q q qq

Como 6

) é divisível por 11, provamos que a tese também é verdadeira. 11(9 10. 2k

q

Logo provamos que 2n+2 6 1 +

11 divide para todo n .3 2 Zn

Questão 12

Considere nF a sequência de Fibonacci. Mostre que

n. .

1 15 51 1F

2 25 5

n n

Um solução:

A sequência de Fibonacci corresponde à recorrência 1 2 1 2

, com 1n n nF F F F F . Temos

então que essa sequência é 1,1,2,3,5,8,13,21,34,.......

Vamos demonstrar a validade de n

. .1 15 51 1

F2 25 5

n n

, que define cada termo dessa

sequência. Vamos utilizar indução finita.

Para n = 1

Page 64: MA21 SOLUÇÕES DAS QUESTÕES DO LIVRO

Temos que 1 1

1. . .

1 5 1 5 1 5 1 51 1 11

22 25 5 5F

Como hipótese, consideremos que nF é válido para n = k.

n = k temos que . .

1 5 1 51 12 25 5

k k

kF

Nossa tese será provar que para n = k+1 , nF também é válida.

Para n = k+1 ,

1 1

1. .

. . . . .

1 5 1 51 12 25 5

1 5 1 5 1 5 1 51 12 2 2 25 5

k k

k

k k

F

Temos que 1 1k k kF F F ,

Somando

1 1

1

1 1

. . . .

. . . . . .

. .

1 5 1 5 1 5 1 51 1 1 12 2 2 25 5 5 5

1 5 1 5 1 5 1 5 1 5 1 51 1 1 12 2 2 2 2 25 5 5 5

1 5 11 125 5

k k k k

k k

k k k k

k k

F F

. . . .

. . . .

. . . .

2 25 1 5 1 51 12 2 21 5 1 55 5

1 5 1 51 2 1 21 1

2 25 1 5 5 1 5

1 5 1 53 5 3 51 12 25 1 5 5 1 5

k k

k k

k k

1. . . .

1 5 1 51 5 1 51 12 22 25 5

k k

kF

Outra solução

Consideremos as progressões geométricas n

n qv , com 0q , que satisfaz à recorrência

1 2 1 2, com 1

n n nx x x x x

.

Temos que

Page 65: MA21 SOLUÇÕES DAS QUESTÕES DO LIVRO

1 2

´ ´́

cujas soluções são

1 5 1 5 e

2 2q q

n n n

q q q

Defina

´ ´́

n e w

n m

n q qv .

Como n

e wnv satisfazem à recorrência 1 2 1 2

, com 1n n nx x x x x

,

n

1 2

então, para todo e reais, a sequência também satisfaz à recorrência.

Agora impomos 1, o que nos dá um sistema de duas equações com as duas

1incógnitas e , cujas soluções são =

5

F

F

n nwv

F

1 e =- .

5

Questão 13

Mostre as seguintes propriedades a respeito da sequência de Fibonacci nF :

a) 2

11

n

i ni

F F

b) 2 1 2

1

n

i ni

F F

c) 2 2 1

11

n

i ni

F F

d) 2

1 1. ( 1)

n

n n nF F F

Resolução.

a) 2

11

n

i ni

F F

Para n = 1 temos que 1 3 1 1F F

Hipótese: Vamos supor que seja verdade para n = k com k natural e maior que 1

1 2 2............ 1k kF F F F

Tese: Vamos demonstrar que a fórmula é verdadeira para n = k+1

1 2 1 3

k+2 1 3 3 1 k+2

............

Substituindo a hipótese no primeiro membro da igualdade temos que:

, o que é verdade pois,

1

F 1 1 F

k k k

k k k k

F F F F F

F F F F

b) 2 1 2

1

n

i ni

F F

Para n = 1 temos:

1 2 1F F

Verdadeiro

Como hipótese vamos supor a validade da fórmula para n = k, sendo k natural maior que 1

1 3 2 1 2.............

k kF F F F

Page 66: MA21 SOLUÇÕES DAS QUESTÕES DO LIVRO

Tese: Vamos demonstrar que a fórmula é verdadeira para n = k+1.

Para n = k+1 temos

1 3 2 1 2 1 2 2.............

k k kF F F F F

Considerando a tese temos que

1 3 2 1 2 1 2 2.............

k k k

hipótese

F F F F F

Substituindo a hipótese, então

2 2 1 2 2k k kF F F

O que mostra a validade da Tese.

c)

2 2 11

1n

i ni

F F

Vamos demonstar a validade desta propriedade utilizando indução finita.

Para n = 1 é válida a propriedade.

2 3 1 2 1 1F F

Como hipótese de indução vamos considerar que essa propriedade é válida para n = k , com k

natural e maior que 1.

Para n = k teremos:

2 4 2 2 1......... 1

k kF F F F

Tese : Vamos demonstrar que a propriedade é válida para n = k+1

Para n = k+1

2 4 2 2 2 2 3......... 1

k k kF F F F F

Trabalhando com a tese

2 4 2 2 2 2 3

hipótese de indução

......... 1k k kF F F F F

Substituindo a hipótese de indução

2 1 2 2 2 3 2 1 2 2 2 311k k k k k kF F F F F F

O que demonstra a validade de nossa tese.

d) 2

1 1. ( 1)

n

n n nF F F

Para n= 2 temos a validade da propriedade.

22

1 3 22. .( 1)1.2 11F F F

Hipótese: Vamos supor que a propriedade é válida para n = k com k natural e maior que 2. Para n = k temos

2

1 1. ( 1)

k

k k kF F F

Tese: Vamos demonstrar que a propriedade também é válida para n = k+1.

Page 67: MA21 SOLUÇÕES DAS QUESTÕES DO LIVRO

Para n = k+1 temos que: 12

2 1. ( 1)

k

k k kF F F

Para demonstrar a validade da tese, vamos inicialmente trabalhar com a hipótese:

2

1 1. ( 1)

k

k k kF F F

Multiplicando os dois membros por (-1) obtemos

12

1 1

2 1

1 1

12 2

1 1 1 1 1 1

12 2

1 1 11

.

. 1 1

. .

.

( 1)

( 1)

( 1)2

( 1)3

k

k k k

k

k k

k

k k k k k k

k

k k kk

k k

F F F

F F F F

F F F F F F

FF F F

Agora iremos trabalhar com o primeiro membro da tese

2 2

2 1 1 1 1 1 1

2 2 2

1 1 1 1 1 1 1

2 2 2 2 2 2 2

1 1 1 1 1 1 1 1 11 1

hipótese de indução

. .

.

2

2 2

3 32

k k k k k k k k

k k k k k k k

k k k k k k k k kk k

F F F F F F F F

F F F F F F F

F FF F F F F F F F F

Substituindo a hipótese de indução na tese, obtemos: 1 12 2

1 1( 1) ( 1)k k

k kF F

Com isso demonstramos a validade da nossa tese.

Questão 14

De quantas formas diferentes podemos cobrir um tabuleiro de 2 x n com peças de dominós que cobrem exatamente duas celas do tabuleiro? Considere um tabuleiro com 2 linhas e n+2 colunas. Para preencher o canto esquerdo do tabuleiro, há duas alternativas: colocar um dominó “em pé” , restando um tabuleiro com 2 linhas e n+1 colunas a preencher( iremos preencher o que falta de xn+1 maneiras) ou colocar dois dominós “deitados” restando um tabuleiro com 2 linhas e n colunas (que poderá ser preenchido de xn maneiras distintas). Logo, o número xn de modos de preencher um tabuleiro 2 x n com dominós 2 x 1 satisfaz a recorrência xn+2= xn+ xn+1, com x1 = 1 e x2 = 2. Esta é a sequência de Fibonacci, logo temos

1 1

1 1 5 1 1 5

2 25 5

n n

nnx F

Page 68: MA21 SOLUÇÕES DAS QUESTÕES DO LIVRO

Questão 15

. Calcular o número de regiões em que o plano é dividido por n retas distintas em cada uma das seguintes situações: (a) as n retas são concorrentes; (b) não existem duas retas paralelas nem três retas concorrentes

a) As n retas concorrentes dividem o plano em 2n regiões. Vamos demonstrar tal fato por indução finita. Para n = 1, ou seja uma reta, o plano fica dividido em 2.1 = 2 regiões. Vamos supor que seja verdade que para n = k o plano fica dividido em 2k regiões ( Hipótese de indução) Vamos verificar que para n = k+1 o plano fica dividido em 2(k+1) regiões. Temos que em um conjunto de k retas, cada reta acrescentada (deve ser uma reta concorrente às outras retas) aumenta mais duas regiões. Ou seja para k retas temos 2k regiões, acrescentando mais uma reta teremos 2k + 2 regiões e 2k+2=2(k+1) que demonstra nossa tese.

c)Fazendo alguns desenhos que satisfazem ao problema verificamos que:

O número de regiões P(n) em que o plano fica dividido por n retas, sendo duas não paralelas e

nem três concorrentes é dado por n(n+1)

P(n)= 12

.

Vamos demonstrar a validade de P(n) utilizando indução finita. Inicialmente vamos considerar n = 1. Vemos facilmente que para uma reta o plano fica dividido em duas regiões.

1.(1+1)

P(1)= 1 22

Hipótese: Vamos considerar que para n = k seja validade P(k) = k.(k+1)

P(k)= 12

.

Número de retas (n) Número de regiões

P(n)

1 2

2 4

3 7

4 11

5 16

Page 69: MA21 SOLUÇÕES DAS QUESTÕES DO LIVRO

Tese: Vamos demonstrar que para n = k+1 seja válida P(k+1).

Considerando que para k retas o plano fica dividido em (k+1).(k+2)

P(k)= 12

. Ao acrescentarmos mais

uma reta ela acrescenta k+1 novas regiões. Observe que a nova reta ao encontrar a primeira reta, ela separa em duas regiões a região em que está, entrando em outra região. Ao encontrar a segunda reta, ela separa em duas regiões a região em que está, entrando em outra reta, e assim sucessivamente até encontrar a n-ésima reta. Assim são obtidas k+1 regiões a mais das que já existiam, logo:

P(k+1) = P(k) + k+1 =

2 2k.(k+1)

1 k+1=2

( 1).( 2)3 21 11 1

2 22

k kkkk kk

. C.q.d

Questão 16

Dizemos que uma figura é enquadrável com régua e compasso, se a partir dela é possível, utilizando apenas régua e compasso, construir um quadrado de mesma área. Prove que:

(a) um triângulo é sempre enquadrável; (b) um polígono qualquer é enquadrável. Sugestão para o item (b): Utilize indução dividindo a figura em triângulos.

a+b+c um triângulo de lados a,b e c. Temos que A= p(p-a).(p-b).(p-c) sendo p= .

2

Seja x = p.(p-a).(p-b).(p-c)

Seja

Podemos construir um quadrado com lado L = h que terá área igual a A= p(p-a).(p-b).(p-c) .

b) Considere P(n): Todo polígono de n lados é enquadrável. Base de indução. P(3) é verdadeira pelo item anterior. Suponhamos que P(k) é válida. Supondo P(k) verdadeira, vamos provar que isto implica em P(k+1) verdadeira. Todo polígono com (k+1) lados, ao traçarmos uma diagonal “conveniente”, é particionado em um triângulo e um polígono com k lados. Sabemos construir um quadrado Q1 que tem área igual a de um triângulo e sabemos construir Q2, um quadrado com área igual a área de um polígono de k lados (pela

X 1

Temos que h2= x . 1

h

h 1

Podemos escrever L2= h . 1

L

Page 70: MA21 SOLUÇÕES DAS QUESTÕES DO LIVRO

hipótese de indução). Seja f o lado de Q1 e g o lado do quadrado Q2. Temos que f2 é a área do quadrado Q1 e g2 é a área do quadrado Q2. Pelo teorema de Pitágoras temos que f2 + g2 = m2 , onde m2 é a área de um novo quadrado com lado m. Questão 17 Dê uma resposta à situação á Observação 6.17. Sugestão: Observe a validade do argumento quando o conjunto A tem 2 elementos. Veja que B e C não se intersectam. Ou seja, o passo indutivo falha de n = 1 para n = 2. O erro se encontra no passo indutivo de n=1 para n=2. Para n=1 é verdade, pois se há uma camisa todas tem a mesma cor. Agora vamos considerar n = 2, ou seja há duas camisas. Não posso afirmar que as duas tem a mesma cor, o que foi considerado como verdadeiro na demonstração na observação 6.17. Vamos tentar demonstrar que se n = 1 é verdadeiro então n =2 também é verdadeiro. Para n = 2 . Consideremos o conjunto B = {b1, b2} Vamos dividir em dois subconjuntos B´ = { b1} e B” ={ b2} , temos que B = B´ U B” é fato que não podemos garantir que B´ ∩ B” ≠

Page 71: MA21 SOLUÇÕES DAS QUESTÕES DO LIVRO

Capítulo 7 – Soluções dos Exercícios 1 – Provar que em todo triângulo a soma dos comprimentos das medianas é menor que o perímetro do triângulo e maior que o semiperímetro deste.

Observe a figura, onde o triângulo ABC tem lados a, b e c e as medianas tem medidas ma, (mediana relativa ao lado a ) ,m b (mediana relativa ao lado b) e mc (mediana relativa ao lado c)

Vamos provar inicialmente que a soma dos comprimentos das medianas de um triângulo é maior que o semiperímetro deste, ou seja:

Somando as desigualdades obtemos:

c.q.d

Agora iremos provar que em um triângulo a soma dos comprimentos das medianas é menor que o perímetro do triângulo. Prolonguemos, por exemplo a mediana mb e a partir do vértice A traçamos um segmento de comprimento a e paralelo ao lado BC que intersectará o prolongamento de mb em D. Faremos processos análogos às medianas mc e ma

Temos que:

2

2

2

b

a

c

a c

b c

a b

m

m

m

Somando as desigualdades teremos: a b c

a b cm m m c.q.d

2

2

2

2

a b c

c

b

c

a b c

Temos

cb

ba

ab

m m m

m

m

m

2a b c

a b cm m m

Page 72: MA21 SOLUÇÕES DAS QUESTÕES DO LIVRO

2 –Os centros de três círculos que não se intersectam estão sobre uma reta. Prove que se um quarto círculo toca de forma tangente os três círculos, então o raio deste é maior que pelo menos um dos raios dos três círculos dados. O caso em que uma das três circunferências é tangente internamente à circunferência de raio R já satisfaz a condição de R ser maior que o raio de pelo menos uma das circunferências. Vamos analisar agora outra situação. Considere a figura em que as três circunferências com centros sobre uma reta t, são exteriores duas a duas e tangentes externas à quarta circunferência. Seja C1 de raio R1, C2 de raio R2 e C3 de raio R3. A quarta circunferência tem raio R e centro C. Considere x o espaçamento entre as circunferências 3 e 2 e y, o espaçamento entre as circunferências 2 e 1

Pela figura, considerando o triângulo de lados R + R3 , R + R1 e R1+x+2R2+y+R3, podemos escrever:

1 2 3 1 3

2

2

2

,

2 2

2 2

2 2

yxR R R R R R

yxR R

R R

R R

CQD.

3 – Dado n inteiro positivo, provar que 1

1 2

1

n

j

n

j n

Vamos utilizar o fato que a média aritmética de um conjunto de números reais é maior que ou igual a média harmônica desses valores.

1 1 1 1.......

21 2 ....

MA MH

nnn

n

, multiplicando a desigualdade por n, escrevendo 1+2+ ....+ n como

n(n+1)/2, obtemos:

Page 73: MA21 SOLUÇÕES DAS QUESTÕES DO LIVRO

1 1 1 2.......

2 1

n

n n

que podemos escrever como

1

1 2

1

n

j

n

j n

c.q.d

4 – A soma de três números positivos é 6. Provar que a soma deseus quadrados não é menor que 12. Sejam a, b e c números reais positivos. Temos que a + b + c = 6. Vamos utilizar o fato de que

MA MQ

2 2 2

2 2 2

2 2 2

2 2 2

3 3

23

43

12

a b c a b c

a b c

a b c

a b c

5 – Determinar as dimensões do paralelepípedo de menor diagonal possível, sabendo que a soma dos comprimentos de todas suas arestas é 12. Considere um paralelepípedo retângulo de dimensões a, b e c, como o comprimento de todas suas arestas é 12, podemos escrever, 4a + 4b + 4c = 12, então a+ b+ c = 3.

Seja d a diagonal, sabendo que d =2 2 2

a b c e usando o fato de que MA≤MQ, teremos que:

2 2 2

2 2 2

2 2 2

2 2 2

3 3

13

13

3

a b c a b c

a b c

a b c

a b c

Como queremos a menor diagonal, tomaremos d = 3 e sabemos que a igualdade MA = MQ, ocorre

quando a=b=c e neste caso teremos a = b= c = 1. 6 – Encontrar todas as soluções positivas do sistema de equações não lineares

Considerando a primeira equação do sistema podemos escrever que a média aritmética desses valores é:

2 2 2

1 2 10.............. 1

10 10MA x x x

e utilizando a segunda equação do sistema, podemos escrever

que

Page 74: MA21 SOLUÇÕES DAS QUESTÕES DO LIVRO

2 2 2

1 2 10

10 10 1

1 1 1 100 10..........

MH

x x x

.

Temos que:

2 2 2

1 2 10......MA MH x x x

Substituindo na primeira equação, obtermos 2

1 110 1 1/ 10x x , mas queremos soluções

positivas, logo

1 2 10

1......

10x x x

7 – Demonstrar que, se 1 2, ,.....,

na a a são números positivos tais que

1 2. ...... 1

na a a

Então (1+a1) . (1+a2) ......... (1+an ) 2n.

Suponhamos por absurdo que (1+a1) . (1+a2) . (1+an) < 2n. Dividindo por 2n, obtemos:

1 2(1 )(1 ) (1 )

. ........... 12 2 2

naa a .

Iremos usar o fato de que MG≤MA. Observe que para todos os ai´s, com i {1,2,..... }n , teremos que

11.

2

i

i i

aa a

, logo:

1 2. ........... 1n

na a a ,

Sabemos que

1 2. ...........

na a a = 1 (hipótese do problema),

Teríamos o fato de 1< 1 , o que é um absurdo. Logo vale a desigualdade (1+a1) . (1+a2) ......... (1+an ) 2n.

8 – Prove que a média geométrica é super-aditiva, isto é, para números não negativos

i i e , 1 i n, tem-sea b

1 1 1

( )n n n

n n ni i ii

i i i

aa b b

Além disso, estude em que condições ocorre a igualdade. Sugestão: Use a desigualdade entre as médias geométrica e aritmética. Temos que MA≥MG, logo

Page 75: MA21 SOLUÇÕES DAS QUESTÕES DO LIVRO

11

11

1 11 1

1 1 1

1

1

(I)

(II)

Fazendo (I) ( )

n

)(

nnii n

ii i ii i

nnii n

ii i ii i

n nn ni ii i n n

i ii i i i i ii i i i

n n ni i

n ni i

i i ii i

n

nii

i

ii

n

n

II

n

aaa ba b

bba ba b

a ba ba b a ba b a b

a ba ba b

a b

ann

1 1 1

1 1 1

1 1

1 ) c.q.d

) )

(( (

n n n n

n n n ni i i n n n

i i in n n

i i iin ni i i

n ni ii i

i i

b a ba b a b

a ab b

9 - Usar o método de indução para provar a desigualdade de Cauchy-Schwarz.

Considere P(n) = 2 2 2 2 2 2

1 1 2 2 1 2 1 2........... ........ . ........

n n n na b a b a b a a a b b b

Vamos demonstrar que P(n) é válida para todo n natural, e para tal utilizaremos a indução finita.

Para n = 1 verificamos que P(n) é válida. Observe :

22 2

1 1 1 1 1 1( )a b a b a b

Hipótese de indução: Suponhamos que P(n) seja verdadeira para n = k, então teremos 2 2 2 2 2 2

1 1 2 2 1 2 1 2........... ........ . ........P(k)= k k k ka b a b a b a a a b b b

Tese:

Vamos provar que para n=k+1 , P(n) é verdadeira.

Sendo n = k+1, temos que

1 1 2 2 1 1 1 1 2 2 1 1

2 2 2 2 2 2 2 2 2 2 2 2 2 2

1 2 1 2 1 1 1 2 1 1 2 1

........... ...........

........... . ........... . ..... . ..... .

k k k k k k k k

k k k k k k

a b a b a b a b a b a b a b a b

a a a b b b a b a a a b b b

Vamos verificar a validade desta última desigualdade.

Considere x, y, z e w números reais positivos. Utilizando o fato de que MG≤MA podemos escrever:

Page 76: MA21 SOLUÇÕES DAS QUESTÕES DO LIVRO

22 2

. 2 . ( 2 .2

2 . ( ).( ) ( ).( )

. . . .

( ) ( ) ( )

xw yzxw yz xw yz xw yz xy zw xw yz xy xw yz zw

xw yz y w x z y w x z

x y z w x y y w

xy zw xy zw

2 2 2 2 2 2 2 2

1 2 1 2 k+1 k+1Basta considerarmos ........... , y = ........... , z = e w= a bx k ka a a b b b

e a desigualdade fica demonstrada.

10 – Para todo λ real 2

1

0( )n

ii ia b

Use este fato para dar outraprova da desigualdade de Cauchy-Schwarz

Podemos escrever:

2 2 2 2 2 2 2

1 1 1 1 1

2. . . ) 2. . 0( ) (n n n n n

i i i i i i iii i i i i

i i a a b b a a b ba b

Fazendo:

2

1

n

ii

v b

, u = 1

.n

i ii

a b

e t = 2

1

n

ii

a

,

Obtemos a desigualdade do segundo grau em . 2

02t u v

Temos que v > 0 e 0 2

2

2

4 4 0

4 4 0

4 4

u vt

u vt

u vt

Então teremos que

2

.

.

u vt

u v t

Que é a desigualdade de Cauchy-Schwarz, com 2

1

n

ii

v b

, u = 1

.n

i ii

a b

e t = 2

1

n

ii

a

,

11 – Use a desigualdade de Cauchy-Schwarz para dar uma prova alternativa da desigualdade entre as médias aritmética e quadrática (ma ≤ mq).

A desigualdade de Cauchy-Schwarz nos diz que 2 2

1 1 1

.n n n

i i i ii i i

a b a b

, fazendo 1,ib a

desigualdade poderá se reescrita como 2

1 1

.n n

i ii i

na a

que equivale a

2 2 2

1 2 21.......... ......... ).(n n

naa a a a a ,

Dividindo por n, os dois membros da desigualdade já que n é um número natural e positivo, podemos escrever:

Page 77: MA21 SOLUÇÕES DAS QUESTÕES DO LIVRO

2 2 2

1 2 1 2......... ........

n n

n n

a a a a a a , logo MA MQ . cqd

12 – Prove que 1 1 1

1 2 22

n n n

i ii i i

i ia b a b

Uma solução

Queremos provar que 1 1 1

1 2 22

n n n

i ii i i

i ia b a b

Temos que 1 1

1 1

2 21 2 22 2

n n

n ni i

i i

i i

i i

a ba b

, que consideraremos a média aritmética de dois

termos.

A desigualdade de Cauchy-Schwarz nos diz que 2 2

1 1 1

.n n n

i i i ii i i

a b a b

,

Usando o fato de que MG MA

2 2 1 1

.1 1 1 1 1

2 21 2 2.2 2

n n

n n n n ni i

i i i ii i i i i

i i

i i

a ba b a b a b

13 – Prove que a4 + b4 + c4 ≥ abc(a + b + c)

Uma solução , considerando a, b e c reais.

Inicialmente vamos considerar o fato de que MQ≥MA

Podemos escrever

2 2 2

22 2 2

22 2 23.( )

3 3

( )( )

3 9

a b c a b c

a b ca b c a b ca b c

,

Desenvolvendo esta última desigualdade concluiremos que a2 + b2 + c2≥ ab+ ac + bc .

A partir desta conclusão podemos escrever a4+b4+c4=(a2)2 + (b2)2+(c2)2≥ a2 b2+ a2 c2+ b2 c2 (I)

Temos o fato de que a2 b2+ a2 c2+ b2 c2 ≥ (ab+ac+bc)2/3. (II)

Para justificarmos a desigualdade II basta observarmos que

2 22 2 2 2 2 2 22 2 2 2( )( ) 2.( ). 22 2

3 3 3

ab acab ac bc ab ac bc bc aba b a c abc b cb c a c

=

2 2 2 2 2 2 2 2 2

.23 3

a b a b b c a bc ab c abc ≤

2 2 2 2 2 2

a b a b b c .

Page 78: MA21 SOLUÇÕES DAS QUESTÕES DO LIVRO

Vamos trabalhar com a desigualdade II, temos que

a2 b2+ a2 c2+ b2 c2≥(ab+ac+bc)2/3 que equivale a

2 2 2 2 2 2

a b a c b c ≥ 2 2 2 2 2 2 2 2 2

.23 3

a b a b b c a bc ab c abc ≥

2 2 2( )( ) IIIabc a b ca bc ab c abc .

Pela desigualdade I temos que

4 4 4 2 2 2 2 2 2

a b c a b a c b c

e a desigualdade III nos fornece que

2 2 2 2 2 2

( )abc a b ca b a c b c

Logo, concluímos que

a4 + b4 + c4 ≥ abc(a + b + c)

Outra solução considerando a, b e c ≥ 0 Inicialmente vamos considerar o fato de que MA≥MG

4 4

4 4 4 4 2 2

4 4

4 4 4 4 2 2

4 4

4 4 4 4 2 2

. .

. .

. .

22

22

22

a ba b a b a b

a ca c a c a c

b cb c b c b c

Somando estas desigualdades 4 4 4 2 2 2 2 2 2

4 4 4 2 2 2 2 2 2

2 2 2 2 2 2

( )

a b c a b a c b c

Ia b c a b a c b c

Utilizando novamente a desigualdade entre as médias aritmética e geométrica podemos escrever: 2 2 2 2

2 2 2 2 2 2 2 2 2

2 2 2 2

2 2 2 2 2 2 2 2 2

2 2 2 2

2 2 2 2 2 2 2 2 2

. .

. .

. .

22

22

22

a b b ca b b c a b b c a b c

a b a ca b a c a b a c a b c

a c b ca c b c a c b c a b c

Somando as desigualdades obtemos

2 2 2 2 2 2 2 2 2

2 2 2 2 2 2 2 2 2

2 2 2 2 2 2( )

2 2 2 2 2 2

( ) II

a b a c b c a bc ab c abc

a b a c b c a bc ab c abc

abc a b ca b a c b c

Page 79: MA21 SOLUÇÕES DAS QUESTÕES DO LIVRO

A desigualdade I nos diz que

4 4 4 2 2 2 2 2 2

a b c a b a c b c

e a desigualdade II nos fornece o fato de que 2 2 2 2 2 2

( )abc a b ca b a c b c

Comparando I e II concluímos que

4 4 4

( )abc a b ca b c c.q.d

14 – Prova que se a≥0, b≥0 e c ≥ 0. Então (a + b)(a + c)(b + c) ≥ 8abc:

Vamos usar o fato de que MA MG

. 2.2

a ca c a c ac

2

b cbc b c bc

Multiplicando as inequações, obtemos: (a+b) . (a+c) . (b+c) ≥ 8.a.b.c c.q.d 15 – Queremos provar que (1+x)n> 1+nx para todo x positivo e n natural , Uma solução

Temos que (1+x)n> 1+nx (1 ) 1nx nx Sendo n natural e maior que um. Considere os números positivos a1, a2, ......an, tais que a1. a2. ...... .an = 1 + nx (produto dos números) e a1+ a2+ .....+.an = n + nx

Podemos dizer que um dos ai, com i {1,2,...n} é igual a 1+nx. Vamos supor que a1 = 1 + nx, logo teremos que a2=a3=........=an = 1. Usando o fato de que MG≤MA,

1 1 1 1 (1 )n

n nnx nnx nx x nx

nx

, como a ocorre para a1=a2=.....=an

E nossos valores não serão todos iguais a um teremos o que queríamos demonstrar (1+x)n> 1+nx Outra solução Agora iremos demonstrar tal desigualdade utilizando indução. (1+x)n> 1+nx Observe que para n = 1 teremos (1+x)1 = 1 + 1.x, ou seja uma igualdade, logo não vale para n = 1. Iremos fazer a nossa indução para todo n natural e maior que 1. Então faremos o teste para n = 2 (1+x)2 > 1+2x , vamos inicialmente desenvolver (1+x)2 = 1 + 2x + x2, como x é positivo, então x2 > 0, logo 1+2x + x2 > 1 + 2x, o que prova a desigualdade para n = 2.

Page 80: MA21 SOLUÇÕES DAS QUESTÕES DO LIVRO

Agora como hipótese, consideremos que para um n=k, com k natural e maior que 1, a desigualdade (1+x)k > 1+kx seja verdadeira . Vamos verificar se a hipótese verdadeira, implica que para n = k + 1 a desigualdade se torna verdadeira (tese). A nossa tese será então para n = k+1 (1+x)k+1 > 1+(k+1)x. A partir da hipótese, podemos multiplicar os dois membros da desigualdade por (1+x), pois se x é positivo, então 1+x também será positivo. (1+x)k > 1+kx (1+x)k+1> (1+kx) (1+x). Agora vamos trabalhar com (1+kx).(1+x) = 1+x + kx + kx2 =

1+(k+1)x + kx2, como kx2 é positivo, temos que 1+(k+1)x + kx2 > 1+(k+1)x, logo (1+x)k+1 > 1+(k+1)x. c.q.d

16- Prove que se a; b; c e d são inteiros positivos, então:

Vamos utilizar o fato de que MH≤MA. Sejam quatro números inteiros positivos a, b, c e d. Temos que:

4

a b c dMA

e

4

1 1 1 1MH

a b c d

, logo

4 1 1 1 1( ).( ) 16

1 1 1 1 4

a b c da b c d

a b c d

a b c d

17 - Prova que se a≥0, b≥0 e c ≥ 0, então

Vamos usar o fato de que MA MG

Consideremos a, b e c três números maiores que ou iguais a zero. Temos que

2

2

ab bcb acacb

2

2

ba aca bcbca

2

2

ac cbc ababc

Somando as desigualdades, obtemos

( )2

2

ab bc aca bc b ac c ab

( )ab bc ac a bc b ac c ab c.q.d

Page 81: MA21 SOLUÇÕES DAS QUESTÕES DO LIVRO

18 - Prove que se x ≥ 0, então 3x3 - 6x2 + 4 ≥ 0. Sugestão: Use a desigualdade entre as médias aritmética e geométrica. Da inequação 3x3 - 6x2 + 4 ≥ 0, podemos escrever 3x3+ 4 ≥ 6x2 Vamos utilizar o fato de que MA≥MG. Considere os monômios 2x3, x3 e 4 Temos que

34

3

3MA x e

6 23 8 2MG x x

Então,

3

2

3 2 3 2

4

3

32

3 6 3 6 04 4

xx

x x x x

19 - Prove que se x≥ 0, então 8

2 43

x x

Vamos utilizar o fato de que MQ≥MG. Considere os monômios 3x e 4, temos que

2 2

2

(3 ) 4

12

MQ

e

MG

x

x

então podemos escrever

2 2 2

2 2

(3 ) 4 9 1612 12 6

x xx x x

Teremos então que 2

2

2

2

2

2

.(4)

: (3)

. .

9 16 12

36 64 48

36 48 64 0

36 96 144 64 0

36 96 64 144

(6 8) 144

6 8 12

82 4

3c q d

x x

x x

x x

x x x

x x x

x x

x x

x x

Page 82: MA21 SOLUÇÕES DAS QUESTÕES DO LIVRO

20 – Sejam C1 e C2 dois círculos concêntricos de raios r1 e r2, respectivamente, com r1 < r2. Sobre o círculo C1 se marcam dois pontos P1 e P2 diametralmente opostos. Deseja-se encontrar o ponto P sobre o círculo C2 que maximiza a soma d(P) = PP1 + PP2.

Pela desigualdade das médias temos que MA < MQ.

22

2

2

2

121 PPPPPPPP

2

.2)(

2

2

2

121

PPPPPPPPPd

22

2

121 .2)( PPPPPPPPPd

21)( PPPPPd MAX , ou seja, o triângulo 21PPP é isósceles.

Pela conclusão anterior temos que o triângulo POP1 é retângulo, portanto:

21

2

1

2PPOPOP então 2

1

2

2

2

1 PPrr

2 2

1 2( ) 2.

MAXd P r r

Page 83: MA21 SOLUÇÕES DAS QUESTÕES DO LIVRO

Capítulo 8 – Soluções dos Exercícios

QUESTÃO 1 Calcule o quociente e o resto da divisão de p(x) por q(x) para os polinômios p(x) e q(x) dados: (a) p(x) = 3x3 - 2x + 1 e q(x) = -7x - 1;

3x3 + 0x2 - 2x + 1 -7x-1

- 3x3 -3/7x2

-3/7x2 - 3/49x + 101/343 quociente

- 3/7x2 – 2x + 1

+3/7x2 - 3/49x

-101/49x+1

+101/49x -101/343

Resto -> 242/343 (b) p(x) = x5 - 1 e q(x) = x - 1;

x5 + 0x4 + 0x3 + 0x2 + 0x - 1 x - 1

- x5 + x4

x4 + x3 + x2 + x quociente

x4 + 0x3

- x4 + x3

x3 +0x2

- x3 + x2

x2 + 0x

- x2 + x

x - 1

-x + 1

Resto 0 (c) p(x) = 3x5 - 2x3 + 1 e q(x) = x2 + x + 1

3x5 + 0x4 - 2x3 + 0x2 + 0x + 1 x2 + x + 1

- 3x5 - 3x4 – 3x3

3x3 - 3x2 - 2x + 5 quociente

- 3x4 - 5x3 + 0x2

+ 3x4 + 3x3 + 3x2

- 2x3 + 3x2 + 0x

+ 2x3 + 2x2 + 2x

5x2 + 2x + 1

- 5x2 - 5x - 5

Resto - 2x - 4

QUESTÃO 2

Encontre os valores de A e B de forma que

Reduzindo a expressão ao mesmo denominador e igualando os coeficientes do 1º e 2º membros, temos:

Page 84: MA21 SOLUÇÕES DAS QUESTÕES DO LIVRO

2111

1

1

1

1)()1(1

1

122222

BBA

BA

A

BA

xx

x

xx

BxAAx

xx

xBxA

xx

x

x

B

x

A

xx

x

Então A=-1 e B=2

QUESTÃO 3

Se os polinômios x2-x+4 e (x-a)2+(x+b) são iguais, encontre a + b. Sendo os polinômios idênticos então iremos igualar os coeficientes do 1º e 2º membros de acordo com os seus expoentes

3414

122112

124

244

2

222

22222

bbba

aaa

baxaxxx

bxaaxxxxbxaxxx

Então a=1 e b=3 QUESTÃO 4

Quais os valores de a e b que tornam iguais os polinômios P1(x) = x2 - x - 6 e P2(x) = (x + a)2 - b?

Sendo os polinômios idênticos então iremos igualar os coeficientes do 1º e 2º membros de acordo com os seus expoentes:

4

25

4

166

2

16

2

112

266

2

2

22222

21

bbbba

aa

baaxxxxbaxxxPP

QUESTÃO 5

A divisão de P(x) por x4 + 1 tem quociente x + 2 e resto 1. Encontre o polinômio P(x). Usando o principio fundamental da divisão teremos:

32)(

122)(

1)2)(1()(

)()()()(

45

45

4

xxxxP

xxxxP

xxxP

xRxQxDxP

QUESTÃO 6

Qual o resto da divisão do polinômio x100 por x + 1? De acordo com o teorema do resto temos que R = P(raiz do divisor), então:

Page 85: MA21 SOLUÇÕES DAS QUESTÕES DO LIVRO

1

)1(

)1(

100

R

R

PR

QUESTÃO 7

Determine o resto da divisão do polinômio p(x) pelo polinômio g(x) = x, onde p(x) = (x - 1)(x - 2) ... (x - n) + b . Usando o teorema do resto teremos:

bnR

bnR

bnR

PR

n

!.)1(

))...(3)(2)(1(

)0)...(30)(20)(10(

)0(

QUESTÃO 8

Mostre que xn - 1 é divisível por x - 1 para todo 1n

Teremos que R= (1)n – 1 , como (1)n = 1 para n maior ou igual a 1, então R = 1 – 1, R= 0 logo xn - 1 é divisível por x – 1. QUESTÃO 9

Faça os seguintes itens: (a) encontre o quociente da divisão de xn+1 - 1 por x - 1

1...)( Então

. que temos Euclidesde algoritmo Pelo

)()1)(...(1 que Sabemos

1

11

xxxxq

(x)).d(x) + rP(x) = q(x

xrxxbxaxx

nn

nnn

(b) utilize a divisão anterior para calcular a soma 1 + x + x2 +x3 + ... + xn dos n primeiros termos de uma progressão geométrica de razão x. Usando a soma de termos de uma progressão geométrica teremos:

1

1

1

1)(.1...1

1112

x

x

x

xxxxx

nnnn

QUESTÃO 10

Determine o valor de a para que o polinômio P(x) seja divisível por x - a, onde P(x) = x3 + (1 - a)x2 + (1 + a)x - 1

12

1

)(

2

2323

aaR

aaaaaR

aPR

Sendo P(x) divisível por x-a então R = 0.

Page 86: MA21 SOLUÇÕES DAS QUESTÕES DO LIVRO

0

1

012

0

2

1

2

a

a

aa

R

QUESTÃO 11

Mostre que o polinômio P(x) = x100 - 2x50 + 1 é divisível por x2 - 1. Se P(x) for divisível por x2 – 1, então ele é divisível por x + 1 e x – 1, pois x2 – 1 = (x + 1)(x – 1). Então teremos: P(1) = 0 e P(-1) = 0

verdade

P

verdade

P

0121

01)1(2)1(

0)1(

0121

01)1(2)1(

0)1(

5010050100

QUESTÃO 12

Mostre que o resto r(x) da divisão do polinômio p(x) por x - s é r(x) = p(s). Na divisão do polinômio p(x) por x – s, teremos quociente de grau n-1, para p(x) de grau n e o resto terá grau 0, logo o resto é uma constante que chamaremos de r. Usando a equação fundamental da divisão teremos:

rxqsxxp )().()(

Tomando x = s, então:

)(

)(.0)(

)().()(

spr

rsqsp

rsqsssp

Logo r = p(s) que é chamado de Teorema de D’Alambert.

QUESTÃO 13

Dado o polinômio p(x) = an xn + an-1xn-1 + ... + a1x + a0 definimos a derivada de p(x) como sendo o

polinômio, p'(x) = nanxn-1 + (n - 1)an-1x

n-2 + ... + 2a2x + a1: Por exemplo, a derivada do polinômio x5 é o polinômio 5x4 e a derivada do polinômio x3+5x2+2x-1 é o polinômio 3x2+10x+2. Usando as informações, calcule: (a) a derivada dos polinômios: (i) x + 1;

1)(' xp

(ii) x4 + 3;

Page 87: MA21 SOLUÇÕES DAS QUESTÕES DO LIVRO

34)(' xxp

(ii) 1 + x + x2 + x3 + ... + xn.

12 ...321)(' nnxxxxp

(b) Sabendo que p(0) = 1, calcule também o polinômio p(x) cuja derivada é (i) x4.

15

1)( 5 xxp

(ii) -x2 + 1.

13

1)( 3 xxxp

(ii) x3 + 2x2 + 3.

133

2

4

1)( 34 xxxxp

(c) Prove que se p(x) e q(x) são polinômios, então (i) (p + q)’(x) = p’(x) + q’(x)

Dado 01

2

2

1

1 ...)( axaxaxaxaxp n

n

n

n

e 01

2

2

1

1 ...)( bxbxbxbxbxq n

n

n

n

Derivando p(x) e q(x) teremos

xbxbxbnxnbxq

xaxaxanxnaxp

n

n

n

n

n

n

n

n

12

2

1

1

12

2

1

1

2...)1()('

2...)1()('

Somando as derivadas:

xbaxbaxbanxbanxqxp n

nn

n

nn )()(2...))(1()()(')(' 1122

2

11

1

Somando os polinômios e derivando a soma dos polinômios temos:

)()(2...))(1()()()'(

)()()(...)()())((

11

1

22

2

11

1

0011

2

22

1

11

baxbaxbanxbanxqp

baxbaxbaxbaxbaxqp

n

nn

n

nn

n

nn

n

nn

Logo teremos que:

)(')(')()'( xqxpxqp

(ii) (pq)’(x) = p’(x)q(x) + p(x)q’(x) QUESTÃO 14

QUESTÃO 15 (a) 1 + x2 + x4 + ... + x2n-2 é divisível por 1 + x + ... + xn-1? Temos uma divisão de duas somas de PGs finitas, usando a fórmula de soma de PG finita e simplificando elas teremos

Page 88: MA21 SOLUÇÕES DAS QUESTÕES DO LIVRO

12kn temosdivisívelser Para

1

1

1

1.

)1).(1(

)1).(1(

1

1

)1).(1(

)1).(1(

teremos,segundo pelo polinômio primeiro o Dividindo

1

1

1

1)(.1...1

)1).(1(

)1).(1(

1

1)(.1...1

12

2

22242

x

x

x

x

xx

xx

x

x

xx

xx

x

x

x

xxxx

xx

xx

x

xxxx

n

n

nn

n

nn

nnn

nnnn

(b) 1 + x3 + x6 + ... + x3n-3 é divisível por 1 + x + ... + xn-1? Temos uma divisão de duas somas de PGs finitas, usando a fórmula de soma de PG finita e simplificando elas teremos

13kn temosdivisívelser Para

1

1

1

1.

)1).(1(

)1).(1(

1

1

)1).(1(

)1).(1(

teremos,segundo pelo polinômio primeiro o Dividindo

1

1

1

1)(.1...1

)1).(1(

)1).(1(

1

1)(.1...1

2

2

2

22

2

12

2

2

3

33363

xx

xx

x

x

xxx

xxx

x

x

xxx

xxx

x

x

x

xxxx

xxx

xxx

x

xxxx

n

n

nnn

n

nnn

nnn

nnnnn

(c) Generalize.

1 entao ...1)(para ndo,Generaliza 2 pknxxxxp ppnpp

QUESTÃO 16

(a) Resolva a equação 20x3 - 30x2 + 12x - 1 = 0, sabendo-se que1/2 é uma de suas raízes. Usando o dispositivo de Briot-Ruffini, temos:

20 -30 12 1

1/2 20 -20 2 0

Então teremos a equação do 2º grau 22020)( 2 xxxp com raízes

10

155;

10

15521

xx

(b) Uma raiz da equação x3 - (2a+1)x2 + a(a+2)x - a(a+1) = 0 é a + 1, ache as outras duas. Usando o dispositivo de Briot-Ruffini, temos:

1 -(2a+1) a(a+2) -a(a+1)

a + 1 1 -a a 0

Então teremos a equação do 2º grau aaxxxp 2)( com raízes

Page 89: MA21 SOLUÇÕES DAS QUESTÕES DO LIVRO

2

4;

2

4 2

2

2

1

aaax

aaax

QUESTÃO 17

Ache os possíveis valores de a Z para que o polinômio a2x4 + 4x3 + 4ax + 7 seja divisível por x + 1. Sendo o polinômio divisível por x+1 então p(-1)=0, logo:

3

1

034

07)1(4)1(4)1(

0)1(

2

1

2

342

a

a

aa

aa

p

QUESTÃO 18

Prove que todo polinômio de grau 1 é irredutível.

l.irredutíve é um grau de polinômio todoque concluimos Assim

0. grau de polinômio um teríamosoutro, do e, um grau de polinômio um temos

lad, um de que,já ,impossível igualdadea tornaria que o ,constantes

polinômios seriam e seja, ou ,0 que teríamosPortanto,

1 e 1 com )().(

que tais polinômios existiriam R,em redutível fosse Se

1. grau de polinômio um Seja

)h(x)f(x) = g(x

h(x)g(x)= gr(h(x)) gr(g(x)) =

= gr(f(x)) gr(h(x)) <= gr(f(x)) gr(g(x)) <xgxf(x) = h

R g(x)f(x), h(x)f(x)

f(x)

QUESTÃO 19

Prove que se f(x) é um polinômio de grau maior ou igual a 2 e possui uma raiz real, então f(x) é redutível.

redutível. é f(x) Portanto

.1 de grau o onde ,forma da escreve se Então

.por divisível é logo ,0 então , de raizuma é Seja

g(x) - a).g(x)f(x) = (x f(x)

x - af(x)f(a) = f(x)a

QUESTÃO 20

Mostre que todo polinômio f(x) de grau ímpar maior ou igual a 3 é redutível.

Se um certo número complexo é raiz de um polinômio com coeficientes reais, necessariamente seu conjugado também será. Logo, havendo raízes complexas, estas serão em número par, existindo, necessariamente, ao menos uma raiz real "a". Sendo, portanto, o polinômio divisível por x - a. Então será redutível. Obs. Se os coeficientes forem complexos, o Teorema Fundamental da Álgebra já garante a existência de raízes, logo o polinômio será redutível.

Page 90: MA21 SOLUÇÕES DAS QUESTÕES DO LIVRO

QUESTÃO 21

(Critério de Eisenstein). Seja f(x) = a0 + a1x +...+anxn um polinômio com coeficientes inteiros. Suponha

que exista um primo p tal que:

Então, f(x) é irredutível sobre Q.

Mostre que os seguintes polinômios f(x) são irredutíveis sobre Q. (a) f(x) = x4 + 2x3 + 2x2 + 2x + 2; a) 2 não divide 1 b) 2 divide 2, 2, 2 e 2 c) 22 não divide 2 f(x) é irredutível sobre Q (b) f(x) = x6 + 15; a) 3 não divide 1 b) 3 divide 0 e 15 c) 32 não divide 15 f(x) é irredutível sobre Q (c) f(x) = x4 + 10x3 + 20x2 + 30x + 22. a) 2 não divide 1 b) 2 divide 10, 20, 30 e 22 c) 22 não divide 22 f(x) é irredutível sobre Q QUESTÃO 22

Determine quais dos polinômios abaixo são irredutíveis sobre Q. Sugestão: Use o critério de Eisenstein. (a) x3 - x + 1 (b) x3 + 2x + 10 (c) x4 - x + 1 O único polinômio irredutível sobre Q é f(x) = x3 + 2x + 10, pois o número primo 2 não divide o coeficiente an , divide os coeficientes a0, a1, ..., an-1 e 22 não divide o coeficiente a0. QUESTÃO 23

QUESTÃO 24

Determine o polinômio p(x) de grau 7 tal que p(1) = p(2) = ...= p(7) = 8 e p(0) = 1:

8)7)...(2)(1(!6

1)( Portando

!6

1

!6.7

78)70)...(20)(10(1

: termosentão ,1)0( Como

8721

:formada ser deve 7 grau de )( polinômio O

xxxxp

AAA

p

) + )...(x-)(x--p(x) = A(x

xp

Page 91: MA21 SOLUÇÕES DAS QUESTÕES DO LIVRO